MOC EXAM Questions for Child Neurology From Practice Test 1

Ace your homework & exams now with Quizwiz!

A 2-week-old boy with congenital heart disease is evaluated for possible seizures. He was born at term, and became hypotensive shortly after birth. He was determined to have an interrupted aortic arch, and has been dependent upon a prostaglandin infusion to maintain systemic blood flow through the ductus arteriosus. He also has dysmorphic features with micrognathia and low-set ears. He has been irritable since birth and now has a 24 hour history of repeated events involving clonic jerking of the extremities, usually on the right side, although the nursing staff also documents left-sided clonic events as well. Serum calcium is abnormally low (ionized calcium 2.8 mg/dl, normal range 4.1 - 5.2 mg/dl). His head CT is normal, but the EEG is markedly abnormal, with low amplitude irregular slowing, and high amplitude multifocal independent spikes. Treatment consisted of intravenous administration of calcium gluconate and phenobarbital. Which of the following is the most likely diagnosis? DiGeorge syndrome (chromosome 22q11.2 deletion syndrome) Down syndrome (trisomy 21) Group B streptococcal sepsis and meningitis Propionic aciduria Prenatal hypoxic-ischemic brain injury

Correct Answer is: DiGeorge syndrome (chromosome 22q11.2 deletion syndrome) In the 1960's and 1970's, neonatal hypocalcemia (and seizures) were commonly encountered as an unintended result of high phosphate levels in cow's milk-derived infant formulas. In current clinical practice, neonatal hypocalcemic seizures are uncommon, and usually result from deficiency of parathyroid hormone due to hypoplasia of the parathyroid glands. This is often associated with various forms of congenital heart disease including the cardiac outflow tract malformations associated with DiGeorge syndrome (chromosome 22q11.2 syndrome). Treatment consists of restoring physiological levels of calcium. It should be recognized that the seizures and encephalopathy associated with hypocalcemia may take several days to resolve after normal calcium levels have been achieved.

A 14 year old girl with a seizure disorder presents because of ongoing seizures. She was initially diagnosed with grand mal epilepsy following her first tonic clonic seizure 12 years of age. Her teachers have expressed concern about brief periods of "tuning out " in the classroom. Her parents tell you she is clumsy and seems jumpy first thing in the morning . Her EEG shows generalized spike-wave discharges. Which of the following anticonvulsants is likely to worsen her seizures? oxcarbazepine clonazepam lamotrigine sodium valproate topiramate

Correct Answer is: oxcarbazepine The patient is experiencing absence seizures with co-existing grand-mal seizures. All of the drugs listed except for oxcarbazepine are used for generalized absence and primary or secondary tonic-clonic epilepsy. Oxcarbazepine and related drugs can exacerbate generalized epilepsy, and are most frequently used for localization-related (focal) epilepsy.

A 9-year-old boy has had chronic ear infections for years necessitating PE tube insertion bilaterally. A year ago, the tubes fell out and he did not get them replaced. He developed ear pain and fever a week ago and his pediatrician gave him amoxicillin, but it has not helped. In fact, the bony bump behind his right ear is now very tender, but he has resisted saying anything to his mother because he does not want her to cancel their planned trip over the holiday break from school. This morning, he awakened with a bad frontal headache and his vision is blurry. He finally tells his mother and she brings him to a local emergency room thinking he must have developed a brain tumor that has invaded the bone behind his ear. The Emergency Medicine physician finds an ill-appearing youngster with a temperature of 38.5⁰C, tenderness over the right mastoid, and bilateral papilledema. The rest of his examination, including a detailed neurological examination, is normal. A CT scan reveals only lack of aeration of the right mastoid air cells and a white area inside of the right lateral venous sinus posteriorly. The Emergency Medicine physician decides to obtain: A lumbar puncture with opening pressure now and an MRI/MRV later An EEG with sphenoidal leads now and a CT angiogram later An MRI of the orbits now and an MRI of the spine later A Humphreys visual field test now and a serum vitamin A level later A CBC and ESR now and a blood culture later

Correct Answer is: A lumbar puncture with opening pressure now and an MRI/MRV later In the antibiotic era, both mastoiditis and lateral cerebral venous sinus thrombosis have become much less common. They do occur, however, and are often associated with an acute, antibiotic-resistant exacerbation of chronic otitis media. Occlusion of the dural venous flow can cause pseudotumor cerebri and a lumbar puncture is both diagnostic and therapeutic. This child's CT scan already demonstrates the thrombosis in the dural venous sinuses on the right and neither the CT scan nor the child's examination demonstrates evidence of focal differential intracranial pressure, so there is no need to obtain an MRI/MRV urgently. Visual fields should be obtained, but are not of emergent importance. Neither an EEG nor an MRI of the spine would be likely to be useful in this child. In a febrile child with systemic spread of a presumed bacterial infection, a blood culture should be obtained expediently. While vitaminosis A can cause pseudotumor cerebri, that is unlikely to be the cause of this youngster's difficulties.

Q40 1.A 13 year old girl presents with gait difficulties, numbness, eye pain and diplopia. You examine her and find visual acuity of 20/30 OS, 20/30 OD, eye pain with movement and severe photophobia, RAPD, decreased sensation of the left face and hand, left lower extremities bilaterally, and to T6 on the left, bilateral upgoing toes, brisk reflexes overall, and motor weakness (4+/5 in the left biceps, triceps, grip, and ileopsoas). You perform an MRI scan of the brain, orbits and spine, shown above. Click here to download the attachment (printed and labeled MS) What is the most likely diagnosis? ADEM MS CIS-TM CIS-ON MDEM 2. This is the first episode of neurological dysfunction for the child described in Question 40. Can you make a diagnosis after only one clinical episode? yes no 3. What additional finding in the child described in Question 40 would provide another diagnostic criterion for MS? Evidence for prolongation of p100 on VEP DWI positive lesions on MRI Somatosensory evoked potentials showing abnormalities Enhancement of lesions on MRI T1-"black holes" on MRI 4. The child described in Question 40 satisfies criteria for MS. Which two secondary complications are frequently seen in children with MS? Severe spasticity and fatigue Urinary problems and depression Depression and fatigue Fatigue and urinary problems Severe gait abnormalities and depression 5. If a therapeutic intervention is chosen at this point for the child described in Question 40, what could you do to manage her acute event? Solumedrol 20-40 mg/kg/d for 3 days IVIG 2 g/kg x1 Plasma exchange Cyclophosphamide 500 mg/m2 x one dose Rituximab 500 mg/m2x one dose

1. Correct Answer is: MS The distribution of the lesions is typical for MS. The child has multiple white matter lesions of the centrum semiovale as well as a typical ovoid lesion of the spinal cord. The third image shows enhancing lesions on T1 weighted imaging. 2. Correct Answer is: yes This child already has lesions dissociated in time and space. As described in the Discussion for Question 39, their distribution is typical of MS. 3. Correct Answer is: T1-"black holes" on MRI A 13 year old child may receive the diagnosis of MS with a single MRI using revised McDonald criteria showing evidence for dissemination in space and time. Enhancement of lesions suffices as evidence for "active" lesions, T1-black holes are regarded as evidence for older lesions, I,e, dissemination in time. As she is presenting with an acute episode, it is necessary to establish the presence of "old" lesions, or T1 hypointense lesions. 4. Correct Answer is: Depression and fatigue Irreversible motor difficulties are rarely seen early on in pediatric MS; irreversible motor problems and gait abnormalities/spasticity will occur on average about 20 years after onset of disease, thus not during childhood. On the other hand, depression and fatigue are very common in this population, occurring in anywhere from 3075% of the population. 5. Correct Answer is: Solumedrol 20-40 mg/kg/d for 3 days Use of pulse steroids at 20-40 mg/kg/d is standard therapy for treatment of acute exacerbations, although no randomized controlled trials have been performed in the pediatric population

Q38 A 5 year old child presents with unusual arm movements. 24 hour continuous EEG does not show epileptiform activity. While in hospital she develops abnormal mouth movements and begins to express extreme fear. She then develops an inability to communicate with others. She develops difficulty sleeping. An MRI scan of the brain is performed. It is within normal limits. LP is normal. HSV is negative. NMDAR testing is sent and is positive. 1. For what associated neoplasm should you search? Medulloblastoma Neuroblastoma Teratoma Oligodendroglioma Thymoma 2. You test the child described in Question 38 for the associated neoplasm. The test returns negative. What can you tell the family about future prognosis? No recurrence is likely; likely return to normal function within 2 years No recurrence is likely; likely return to normal function within 6 months No recurrence is likely; but likely will continue to have neurological abnormalities Recurrence likely, with need for ongoing therapy; no return of function lost

1.Correct Answer is: Teratoma This is most likely to be found in the adult population, but a small proportion of children have associated neoplasm (9% of girls <14 years of age). In the adult population, the first descriptions of this syndrome were in association with ovarian teratoma 2.Correct Answer is: No recurrence is likely; likely return to normal function within 2 years Risk of recurrence without associated neoplasm was found to be approximately 10% in a mixed adult and pediatric cohort and 25% in a solely pediatric cohort.

You are asked to see an 8 month old with the question of whether he might have cerebral palsy. He was born vaginally at term to a healthy 33 year old mother with one prior successful pregnancy. She was healthy during the pregnancy but was told the placenta was "small and old looking". The baby was discharged on day 3 following observation for temperature instability on day 1. He received regular pediatric care and was noted to be 'a little floppy but strong' at the visits through 6 months. He was referred due to maternal concern that he is not meeting motor milestones as quickly as his sister did. Mother reports that he is 'left handed like his father' since he began reaching for toys. Which of the following are 'red flags' for possible cerebral palsy: A. Abnormal placenta B. Low tone in early infancy C. Hand preference in first year of life D. A, B and C

Correct Answer is: A, B and C Abnormal placental findings including inflammation, calcification and small size are risk factors for cerebral palsy. Risk factors for cerebral palsy at term include a history of placental abnormalities, birth defects, low birth weight, meconium aspiration, instrumental⁄emergency Caesarean delivery, birth asphyxia, neonatal seizures, respiratory distress syndrome, hypoglycemia, and neonatal infection. Low tone in the first year of life can be an indicator of cerebral palsy which might give way to increased tone in later infancy. Hand preference before the age of one can be abnormal and is a red flag for possible hemiplegia

A 4-year-old girl has a tonsillectomy and adenoidectomy performed. The operation proceeds without untoward event and she is moved to the recovery room. She is easily extubated and slowly begins to awaken. When she is largely awake, gags and vomits twice. The nurse pages the ENT surgeon, who tells her to give the child some intravenous metoclopramide. The vomiting does not recur, but she suddenly begins to have episodes lasting 1-3 minutes each of tonic, conjugate eye deviation up and to the right and tonic mouth and tongue twisting to the left. Although she is mute when they occur, she seems to be awake and quite upset during these episodes; she cries inconsolably between them. The episodes are now occurring every few minutes. The appropriate intervention for this child would include: Administration of fosfenytoin Performance of a stat EEG Administration of diphenhydramine Performance of a stat lumbar puncture Administration of more metaclopramide

Correct Answer is: Administration of diphenhydramine This child is experiencing oculogyric crises, most likely related to the administration of metoclopramide. Oculogyric crises occur most often as a medication-related side effect and are most commonly seen with phenothiazine administration. They are characterized by tonic eye deviation, most often up and laterally, and can be accompanied by tongue protrusion or deviation, twisting of the mouth, and/or dystonia of the neck muscles. Drooling, anxiety, irregular breathing, and mutism can also be seen. Between paroxysms, the patient is often irritable and/or fatigued. Treatment for oculogyric crises includes the acute administration of anticholinergic medication; in young children, diphenhydramine is the most common agent used. Anticonvulsants, and particularly diphenylhydantoin and fosfenytoin, can exacerbate this condition, as would additional doses of metoclopramide; these are therefore contraindicated

Which of the following is true regarding pediatric headache? The prevalence increases throughout childhood. In the pre-pubertal years the prevalence of migraine is higher in males than females Migraine can result in significant disability in pediatric patients Your Answer All of the above

Correct Answer is: All of the above Discussion: Various population-based studies have shown the overall estimated prevalence of headache in children and adolescents is approximately 60%. The prevalence of migraine specifically is 7.7%. Overall headaches especially migraines are more common in females. Post pubertal females are more likely to have chronic migraine than males.

A 30-month-old girl is being evaluated for unexplained spells. She seemed to be developing normally until episodes of unusual eye movements were noted around 2 months of age, consisting of nystagmus, sometimes affecting only one eye. There was no alteration of consciousness. At 6 months of age she developed episodes of flaccid weakness, affecting one side of the body or the other. These events occurred only while awake, and were sometimes provoked by bathing. Weakness would last for several hours before recovery to baseline. If she slept after onset of the episode, she would always awaken without the unilateral weakness. Therapeutic trials of medications for epilepsy and migraine have proven ineffective. On physical examination, she is poorly socialized and has only a few words. She appears to be clumsy and mildly ataxic, despite the fact that she is on no medications at this time. Her eye movements are normal. She was mildly hypotonic, but showed no lateralizing features on her motor exam. Which of the following is the most likely diagnosis? Alternating hemiplegia of childhood Hypokalemic periodic paralysis Mitochondrial encephalopathy with lactic acidosis and stroke-like syndrome Narcolepsy with cataplexy Simple partial epilepsy

Correct Answer is: Alternating hemiplegia of childhood Alternating hemiplegia of childhood (AHC) is a rare disorder with peculiar symptomatology. The correct diagnosis is usually delayed for several years relative to the onset of first symptoms. AHC is in some ways an inadequate descriptive name for the syndrome: 1) the first symptoms are usually episodes of abnormal eye movements, occurring in early infancy, 2) unilateral flaccid hemiplegia occurring independently on each side is the classic expectation, but many patients have simultaneous involvement of both sides of the body for at least some events (double hemiplegia), with episodes characterized by quadriplegia, dysphagia, dysarthria, and possibly respiratory compromise, 3) some episodes may consist of unilateral dystonia, rather than flaccid weakness, and 4) virtually all patients develop persisting co-morbid symptoms in between hemiplegic episodes, with developmental delay, ataxia, and/or epilepsy. Sleep predictably relieves the hemiplegic episodes, and induction of sleep is the most effective way of acutely treating an event. Flunarazine can be effective in some patients to reduce the frequency and severity of attacks

You are on-call for inpatient Neurology consultations and are asked to see a 12-year-old boy in the pediatric intensive care unit who was brought in by ambulance after he collapsed while playing soccer at school. His gym teacher told the EMTs who picked him up from the soccer field that the boy seemed fine before the episode and although he had headed the ball once or twice during practice, nothing unusual seemed to have happened until he suddenly collapsed. The boy ran down the field with his teammates, but did not make contact with the ball, suddenly stopped, grimaced, held his head with both hands crying out, and fell to the ground. He seemed to jerk all over 3 or 4 times and then lay unconscious, intermittently extending both arms and arching his back. He has been unconscious ever since. His mother tells you that, over the four weeks before this happened, he had three episodes of headache and neck stiffness upon awakening, but all three times, the symptoms went away shortly after he got out of bed, so she thought it might have something to do with the new pillow she had bought him a month or so ago. She also tells you that her mother died a few years ago of a glioblastoma. The boy's father and paternal grandmother have polycystic kidney disease, but no one she knows of except her mother "has had anything wrong with their brain." You tell her the boy most likely had an intracranial hemorrhage because of: A brain tumor Head trauma An arteriovenous malformation An infection An aneurysm

Correct Answer is: An aneurysm This boy sustained an intracranial hemorrhage with a subarachnoid component after several episodes during the prior month of sentinel bleeding. His family history is remarkable for one elderly individual with a glioblastoma, generally not a hereditary condition, and two relatives with polycystic kidney disease, a disorder which is genetically linked to the occurrence of cerebral aneurysms

Parents of a 7 year old boy with Duchenne Muscular Dystrophy bring their child in for a follow up visit. They are concerned that their child is having difficulty with academics in the 2nd grade. He is having difficulty with all subjects. The teacher is refusing to recommend psychoeducational testing at this time because many children in her class are having difficulty and he is not failing any subjects. The parents would like your opinion on whether there can be cognitive involvement in a child with DMD. What would be the proper response? Cognitive involvement is uncommon in DMD All children with DMD have intellectual disability Attention Deficit Hyperactivity Disorder is prevalent in DMD The mean IQ in children with DMD is 100 Delayed language development is uncommon

Correct Answer is: Attention Deficit Hyperactivity Disorder is prevalent in DMD Cognitive involvement is common in DMD. Of patients with DMD, 30% have intellectual disability. The average patient's IQ is 85, one standard deviation below the mean. Verbal IQ is more affected than performance IQ, and delayed language development is common. There is a prevalence of comorbid neurodevelopmental /psychiatric disorders is more prevalent in DMD: Attention deficit hyperactivity disorder is 10-15%, Autism spectrum Disorder 3-6%, and obsessive compulsive disorder is 5%.

An 8 year old boy has clusters of nocturnal motor events which happen shortly after he falls asleep. Occasionally, his parents observe him to shiver before the events. During the events, he has tonic stiffening of the limbs and the body with superimposed clonic movements. He tells you he is aware throughout these events, and they frighten him. His brain MRI and interictal EEG are normal . The most likely diagnosis is: Temporal lobe epilepsy Pseudoseizures Autodomal dominant nocturnal frontal lobe epilepsy Parasomnias Simple partial seizures with secondary generalization

Correct Answer is: Autodomal dominant nocturnal frontal lobe epilepsy Autosomal dominant nocturnal frontal lobe epilepsy (ADNFLE) is the first distinctive syndrome of focal epilepsy with single gene inheritance to be described. Onset is mainly in late childhood at 7-12 years of age. ADNFLE manifests with frequent (nearly every night) clusters of brief (20-50 sec) nocturnal motor seizures, with hyperkinetic/dystonic features or tonic manifestations. Motor symptoms consist of thrashing hyperkinetic movements and dystonic posturing (shoulder or pelvic thrashing movements, bipedal and fencing postures) or tonic stiffening of the limbs and the body, often with superimposed clonic components.. Two-thirds of patients experience a non-specific aura of somatosensory, sensory, psychic, and autonomic symptoms. Consciousness is usually preserved. Sleep is abruptly interrupted, but immediately resumes with the end of the seizure. Seizures are most likely to occur in the hypnagogic stage of sleep or shortly before awakening. Brain imaging is normal. Interictal and often ictal EEG are usually normal and unhelpful. On video-polysomnographic EEG, ictal epileptiform abnormalities occur predominantly over frontal areas in 32% of patients, and the EEG shows ictal rhythmic, slow wave activity over anterior areas in another 47% of patients.

A 7-year-old girl presents to the urgent care after she was seen wandering around the playground, and then vomited at school. The teacher who is present with her at the urgent care described her as clumsy, which was uncharacteristic for her. On examination, she is pale and complains of nausea. She is also complaining of a headache in the back of her head, which the teacher says started after arrival in the urgent care. When the mother arrives, she states that the child did this in a similar fashion twice before, over the past 4 months. Mother describes that the child is generally is back to baseline after an extended nap. Which of the following is the most likely diagnosis? Abdominal migraine Acute confusional migraine Acute post-traumatic migraine, after unwitnessed fall on the playground Basilar migraine . Cyclic vomiting syndrome

Correct Answer is: Basilar migraine Basilar migraine is one of the complicated migraine variants, likely the most common, said to represent 3-19% of all migraine. The mean age of onset is 7 years. The diagnosis usually requires the presence of 2 or more clear signs and symptoms of posterior fossa involvement. These include: vertigo, nausea or vomiting, ataxia, visual field deficits, diplopia, tinnitus, hearing loss, confusion, dysarthria, weakness, syncope, with the first 4 being the most common. The typical child will begin with the posterior fossa symptoms such as intense dizziness, vertigo, ataxia and diplopia, which can last for minutes to an hour, and then are followed by the headache phase. The headache is often occipital in location

A 14-year-old girl was helping her mother clean up the basement. She bent over to pick up a blanket from the floor. As she stood up, she banged her head into the bottom of a shelf that extended out from the wall. She did not lose consciousness and the pain of the impact was gone within a minute. A week later, she began having daily headaches. The headaches were bilateral, pulsatile, and diffuse with a frontal predominance. When she awakened each morning, they were almost gone. But by lunchtime, they were moderate in intensity and she had difficulty concentrating at school. When she came home from school each day, her mother noted that she seemed irritable, which was very unusual for her. She sometimes went to bed and slept for a couple of hours before attempting to do her homework. Most of the time, taking a nap made the headaches bearable enough that she could complete her assignments. Her past medical history is remarkable for headache only just before she gets her period each month. Her younger brother gets carsick and her mother gets "sinus headaches" that are preceded by zigzag lines in front of her eyes and accompanied by nausea. The girl's general and neurological examinations are normal. What would be the most useful next step? Order an MRI scan of the head Begin a therapeutic trial of amitriptyline Have a school psychologist evaluate the girl for school phobia Do a lumbar puncture Refer the girl to an otolaryngologist for sinus evaluation

Correct Answer is: Begin a therapeutic trial of amitriptyline Migrainous headache that occurs within days-to-weeks of a minor head injury in a child with the genetic susceptibility to migraine is called post-traumatic migraine. This girl's mother's headaches are not sinus headaches, as they are preceded by a scotoma, accompanied by nausea, related to motion sickness, and unrelated to fever, cough, or nasal discharge. The girl herself has had menstrual headache, also a migraineassociated syndrome. Amitriptylline is among the most commonly used and efficacious agents for post-traumatic migraine

A 16-year-old girl whose parents are friends of yours is evaluated for possible seizure activity. She was observed to have episodes of staring, morning jerks, and had one convulsion. A diagnosis of juvenile myoclonic epilepsy (JME) was made and confirmed on EEG. Additional testing revealed that she tested positive for marijuana and cocaine. She was also found to have genital herpes. The patient asks that her parents not be told of her drug or sexual activities. Which of the following is the most appropriate management? Begin treatment for JME with valproate and inform her parents of the social problems described. Begin treatment for JME with valproate and consult your state law on disclosing information to parents. Begin treatment for JME with lamotrigine and inform her parents of the social problems described. Begin treatment for JME with lamotrigine and consult your state law on disclosing information to parents. Refrain from treatment of her JME until social problems described are adequately resolved.

Correct Answer is: Begin treatment for JME with lamotrigine and consult your state law on disclosing information to parents. The patient presents with a relatively obvious epileptic syndrome. Given her sexual proclivities, the use of valproate should be avoided because of its potential teratogenic effect. The main issue has to do with what to tell the parents. Child neurologists might disagree as to whether we think the patient's welfare is best served by discussing the problems with the parents or abiding by the secrecy requested by the patient. However, we need to be aware laws vary from state to state and need to be carefully addressed before decisions are applied.

A 12 year old boy with a known diagnosis of mild left hemiparetic cerebral palsy presents to your office after a growth spurt for loss of range of motion in his left elbow. Prior to recent months he has been independent with household chores, but now is having difficulty extending the elbow to the point he is unable to push the lawn mower. He does not have other functional motor deficits. On exam he has passive and active range of motion of left elbow extension to only 90 degrees as compared to only minus 20 degrees from full extension recorded 8 months ago. He was discharged from physical therapy over a year ago and is fairly faithful to his stretching routine. Which of the following treatment options would be most appropriate? Oral tizanidine Botulinum toxin type A injection Oral diazepam Intrathecal baclofen

Correct Answer is: Botulinum toxin type A injection This boy has localized spasticity. Botulinum toxin type A has been identified as an effective and generally safe treatment (Level A) for this type of spasticity. Although oral diazepam or tizanidine can be used for short and long term tone management, they are more appropriate for generalized spasticity. His mild focal symptoms do not warrant intrathecal management.

A 3-year-old boy is being evaluated for motor delay. He was born at 38 weeks gestation following an uneventful pregnancy. He was reportedly normal at birth and in the first few months of life, but thereafter developed macrocephaly and was delayed in his milestones. His family history is unremarkable. On examination, he is hypotonic and his head circumference is above the 95th percentile. A brain MRI scan is obtained and T2 weighted samples are shown. Which of the following is the most likely diagnosis? Adrenoleukodystrophy Canavan disease Krabbe disease . Metachromatic leukodystrophy Vanishing white matter disease

Correct Answer is: Canavan disease The triad of hypotonia, macrocephaly, and head lag in an infant after the age of three to five months should raise the suspicion of Canavan disease. Diagnosis of Canavan disease in symptomatic individuals relies upon demonstration of very high concentration of N-acetyl aspartic acid (NAA) in the urine. NAA concentration is also elevated in the blood and cerebrospinal fluid (CSF) of children with Canavan disease. In the brain, the elevated NAA can be visualized by 1H MRS as a single peak with a resonance of 2.01 ppm on the spectrum.

Co-morbidities associated with childhood headache include all of the following except: Obesity Asthma Sleep Disorders Chronic motor tic disorder

Correct Answer is: Chronic motor tic disorder Discussion: Multiple co-morbidities have been shown to be associated with pediatric headache. Hershey et al in 2008 showed that obesity was associated with headache frequency and a reduction in weight loss could contribute to reduced headaches over time. Obstructive sleep apnea has been shown to contribute to childhood headaches, particularly morning headaches in some cases. Assessing for comorbidities such as depression, asthma and obesity are important if a daily preventative medication is being considered.

A 12-year-old girl fell in the school yard. She did not break the skin anywhere, but she twisted her right ankle so that it was mildly painful all day. By that night, it was a bit swollen and she held an ice compress against it on and off all evening. Her symptoms remitted over the next couple of days. A week later, she noticed the onset of episodes of redness, heat, and intense burning pain over the whole lower 1/3 of her right leg and foot that last for a few minutes to a few hours, and then remit completely until the next episode. This girl is likely to have: Cellulitis caused by MRSA Erythema multiforme Pellagra Complex regional pain syndrome Erythromelalgia

Correct Answer is: Complex regional pain syndrome The preceding history of minor trauma and episodic nature of the color and temperature change and pain make this likely to be complex regional pain syndrome. Erythromelalgia usually occurs in older individuals and is not preceded by injury. Pellagra involves desquamation and usually there is an identified risk factor for vitamin deficiency. Erythema multiforme has a characteristic pattern on the skin and is usually not this focal or lateralized. Finally, cellulitis does not produce episodic symptoms

A mother brings in her child who is 24 months of age and is not walking. On examination you note that the child is not babbling, does not have a pincer grasp, sits up but does not crawl. The child cannot mimic building a tower of two blocks. The MRI of the brain is normal. Which genetic test has the highest diagnostic yield in a child with unexplained global developmental delay/intellectual disability? Current generation oligonucleotide-based chromosome microarray Test for Fragile X FISH Karyotype MECP2

Correct Answer is: Current generation oligonucleotide-based chromosome microarray The child above has global developmental delays. Children under the age of 6 are considered to have global developmental delays if they have delays in at least two domains (speech/language, social, fine motor or gross motor) that are 2 SD below the mean in comparison to their age related peers. It is recommended that genetic testing be performed on the child with GDD/ID. Microarray is the genetic test with the highest diagnostic yield in the child with unexplained GDD/ID. Karyotyping and FISH testing have a lower sensitivity. Fragile X may be considered in a male with GDD/ID. MECP2 may be considered in a female with GDD.

A 5 year old is referred to your office by the kindergarten teacher. He is having difficulty with his peers. His mom states the teacher told her that he does not interact with others. He only likes to talk about and play with the video game Minecraft. He does not understand peer teasing. He does not understand emotional cues. His mom states that he has a great vocabulary and is doing well academically. In the office he does not make eye contact, has limited facial expression and does not have joint attention. He cannot engage in a back and forth conversation but tells you about Minecraft. You consider whether he fulfills the criteria for an Autism Spectrum Disorder. Which one of the following is not included in the DSM V Criteria for Autism Spectrum Disorder? Deficits in social-emotional reciprocity Deficits in nonverbal communicative behaviors used for social interaction Deficits in developing and maintaining relationships, appropriate to developmental level Stereotyped repetitive movement, speech, or use of objects Delays in language

Correct Answer is: Delays in language The DSM V that was published in 2013 revealed changes in the criteria for ASD. The criteria no longer include language delays. Diagnostic Criteria include: A. Persistent deficits in social communication and social interaction across multiple contexts, as manifested by the following, currently or by history (examples are illustrative, not exhaustive, see text): 1. Deficits in social-emotional reciprocity, ranging, for example, from abnormal social approach and failure of normal back-and-forth conversation; to reduced sharing of interests, emotions, or affect; to failure to initiate or respond to social interactions. 2. Deficits in nonverbal communicative behaviors used for social interaction, ranging, for example, from poorly integrated verbal and nonverbal communication; to abnormalities in eye contact and body language or deficits in understanding and use of gestures; to a total lack of facial expressions and nonverbal communication. 3. Deficits in developing, maintaining, and understanding relationships, ranging, for example, from difficulties adjusting behavior to suit various social contexts; to difficulties in sharing imaginative play or in making friends; to absence of interest in peers. Specify current severity: Severity is based on social communication impairments and restricted repetitive patterns of behavior B. Restricted, repetitive patterns of behavior, interests, or activities, as manifested by at least two of the following, currently or by history (examples are illustrative, not exhaustive; see text): 1. Stereotyped or repetitive motor movements, use of objects, or speech (e.g., simple motor stereotypies, lining up toys or flipping objects, echolalia, idiosyncratic phrases). 2. Insistence on sameness, inflexible adherence to routines, or ritualized patterns or verbal nonverbal behavior (e.g., extreme distress at small changes, difficulties with transitions, rigid thinking patterns, greeting rituals, need to take same route or eat food every day). 3. Highly restricted, fixated interests that are abnormal in intensity or focus (e.g., strong attachment to or preoccupation with unusual objects, excessively circumscribed or perseverative interest). 4. Hyper- or hyporeactivity to sensory input or unusual interests in sensory aspects of the environment (e.g., apparent indifference to pain/temperature, adverse response to specific sounds or textures, excessive smelling or touching of objects, visual fascination with lights or movement). Specify current severity: Severity is based on social communication impairments and restricted, repetitive patterns of behavior C. Symptoms must be present in the early developmental period (but may not become fully manifest until social demands exceed limited capacities, or may be masked by learned strategies in later life). D. Symptoms cause clinically significant impairment in social, occupational, or other important areas of current functioning. E. These disturbances are not better explained by intellectual disability (intellectual developmental disorder) or global developmental delay. Intellectual disability and autism spectrum disorder frequently co-occur; to make comorbid diagnoses of autism spectrum disorder and intellectual disability, social communication should be below that expected for general developmental level. Note: Individuals with a well-established DSM-IV diagnosis of autistic disorder, Asperger's disorder, or pervasive developmental disorder not otherwise specified should be given the diagnosis of autism spectrum disorder. Individuals who have marked deficits in social communication, but whose symptoms do not otherwise meet criteria for autism spectrum disorder, should be evaluated for social (pragmatic) communication disorder. Specify if: With or without accompanying intellectual impairment With or without accompanying language impairment Associated with a known medical or genetic condition or environmental factor Associated with another neurodevelopmental, mental, or behavioral disorder With Catatonia

A 6-day-old baby girl is being evaluated in the NICU for possible seizures. She was born at 34 weeks gestation and was small for gestational age. She has had feeding difficulties and has received phototherapy for jaundice. On examination, she is noted to have microcephaly, hepatosplenomegaly, and a purpuric rash. Head ultrasound shows ventriculomegaly. Routine laboratory studies reveal elevated aminotransferases, direct and indirect hyperbilirubinemia, thrombocytopenia, and hemolytic anemia. Which test is most likely to confirm this patient's diagnosis? Bone marrow biopsy Head CT scan Ophthalmologic examination Rapid plasma reagin (RPR) test Urine CMV culture

Correct Answer is: Urine CMV culture Cytomegalovirus is a ubiquitous member of the herpesviridae family. Primary infection in immunocompetent adults is usually asymptomatic; by contrast, congenital infection is the most prevalent infection-related cause of congenital neurologic impairment. Vertical transmission—during pregnancy through the placenta, during delivery through cervical secretions or blood, or postnatally through breast milk—occurs in 30% of cases; however, not all infected fetuses are affected. Infants who are symptomatic at birth (10%) are typically small for gestational age with feeding problems, ocular problems (chorioretinitis, retinal scars, optic atrophy, central vision loss), sensorineural hearing loss, organomegaly, thrombocytopenia, jaundice, microcephaly, seizures, and abnormal neurologic examinations. Neuroimaging may reveal periventricular cysts and calcifications, ventriculomegaly, vasculitis, hydranencephaly, and/or neuronal migration abnormalities. Perinatal mortality for symptomatic newborns ranges from 2-30%, with survivors having poor neurodevelopmental prognoses. Infected infants who are asymptomatic at birth are still at risk for neurodevelopmental issues, such as learning disabilites and attention deficit hyperactivity disorder. In addition, 5-15% of these infants will develop sensorineural hearing loss, which is typically bilateral and progressive.

Epilepsy syndromes can present in infancy and childhood with co morbid intellectual disability. Genetic defects have been identified in several syndromes. Which epilepsy syndrome-gene defect pair is incorrect? Severe myoclonic epilepsy of childhood (SMEI) - SCN1A X-linked West Syndrome - ARX Rett syndrome variant with intractable epilepsy - CDKL5 Glut1 deficiency - SLC2A1 Early Infantile Epileptic Encephalopathy With Suppression-burst - MECP2

Correct Answer is: Early Infantile Epileptic Encephalopathy With Suppression-burst - MECP2 A number of distinct clinical syndromes of pediatric genetic epilepsy have been described and linked to specific gene defects. Phenotypes may include, in addition to epilepsy, variable degrees of intellectual disability, elements of autism spectrum disorders, other psychiatric disorders, and motor impairment. Severe myoclonic epilepsy of infancy (SMEI) is characterized by early-onset generalized febrile seizures, followed by very frequent afebrile generalized tonic-clonic seizures (GTCS), myoclonic seizures, absences and focal seizures, with stagnation of psychomotor development, intellectual disability and motor impairment. The main genes involved in these conditions encode sodium channel subunits, in particular SCN1A, which is mutated in 61-87% of SMEI. West syndrome is a common severe infantile epileptic syndrome characterized by typical brief tonic spasms, a profoundly abnormal electroencephalography pattern called hypsarrhythmia, and intellectual disability. Early infantile epileptic encephalopathy with suppression-burst (EIEE) is the earliest infantile epileptic syndrome, often evolving to West syndrome. Mutations in the X chromosome genes ARX and CDKL5 are found in some X-linked West syndrome cases (ISSX1 and ISSX2). ARX (Aristaless-related homeobox) is a transcription factor involved in brain development. It is frequently mutated in a spectrum of X chromosome-linked phenotypes characterized by intellectual disability as their cardinal feature. CDKL5 mutations are associated with West syndrome, intellectual disability, and ASD, and also with a Rett syndrome variant that is characterized by infantile spasms and intractable seizures occurring before the appearance of the typical features of Rett syndrome (Hanefeld variant). Altered glucose transport to the brain due to mutations in the GLUT1 transporter gene SLC2A1 causes epilepsy of variable severity and comorbidities.. Classic GLUT1 deficiency is associated with very low cerebrospinal fluid (CSF) glucose concentrations and causes a severe early-onset (<1 year of age) metabolic encephalopathy characterized by movement disorders, epilepsy, and mental retardation, typically associated with de novoSLC2A1 mutations

A 6-year-old girl presents to the office for evaluation of language difficulties. She was born full-term by normal spontaneous vaginal delivery following an uncomplicated pregnancy. She walked at 9 months, said single words at 11 months, and combined words at 20 months. At age 5, her speech became progressively more telegraphic, and she began indicating her needs by pointing or by using sign language. On neurologic examination, she is able to follow simple commands. She has word-finding difficulties in spontaneous speech and on naming tasks and is unable to repeat short phrases. The remainder of her neurologic examination is normal. Formal audiometric testing is unremarkable. Her parents state that she occasionally stares off into space for 30 seconds to 1 minute. She has never had a convulsive seizure, nor does she have a family history of epilepsy. An EEG is most likely to show which of the following patterns? Bilateral periodic epileptiform discharges (BiPEDs) Continuous, diffuse, polymorphic theta and delta slowing Electrical status epilepticus in slow wave sleep Hypsarrhythmia with periods of electrodecrement Normal waking and sleep

Correct Answer is: Electrical status epilepticus in slow wave sleep In 1957, William L. Landau and Frank R. Kleffner reported a "syndrome of acquired aphasia with convulsive disorder in children" and postulated that persistent convulsive discharges were causing a functional ablation of brain areas concerned with language. Since that time, it has become apparent that there are several epileptic encephalopathy syndromes of childhood that are characterized by deterioration of one or more cognitive functions with or without motor, behavioral, and/or psychomotor decline in association with an EEG that shows strong activation of epileptic activity during sleep. At this point, electrical status epilepticus of slow wave sleep (ESES) and continuous spikes and waves during slow wave sleep (CSWS) are considered synonymous terms, and Landau-Kleffner syndrome (LKS) a distinct presentation of ESES/CSWS in which acquired aphasia is the core symptom. LKS typically presents with language disturbance in normally developing children aged 3 and 8 years. Some have argued that the language disturbance is actually a verbal auditory agnosia (i.e. inability to comprehend spoken language in the absence of hearing difficulties). Over time, affected children experience a gradual deterioration in verbal comprehension and production, eventually leading to failure to respond to non-verbal sounds and mutism. Seizure semiology varies, and 20-30% of patients do not exhibit clinical seizure activity at all. There is no clear relationship between the severity of the seizures and the severity of the language deficits. EEG shows predominantly bilateral posterior temporal spikes or spike-wave discharges that are activated by sleep. The clinical seizures are usually easy to control, with spontaneous resolution occurring in mid-adolescence. There are several treatment options, including corticosteroids, levetiracetam, benzodiazepines, and valproic acid. Language function improves when the active phase of spike-wave discharges resolves; however, if the aphasia begins in early childhood and/or persists for more than 1-2 years, long-term language deficits are almost universal.

Emily is 17 years old and presents to the emergency department with a cluster of 8 convulsive seizures in 12 hours, with each seizure lasting for 3 minutes. She gets an aura she cannot describe for few seconds before each seizure. Each seizure is characterized by head and eye deviation to one side, and there was no obvious trigger for this cluster. The history of Emily's illness includes clusters of convulsive seizures every 3 months. She also has focal dyscognitive seizures , but she is not certain whether she has ever had seizures of other types. She does know that, when she was 9 months old, she began having clusters seizures whenever she ran a fever. Emily has mild intellectual disability, is said to have an autism spectrum disorder with obsessive features. She is currently taking daily levetiracetam, valproate, and lamotrigine. Her constellation of symptoms and signs suggest that Emily has: Dravet Syndrome Juvenile myoclonic epilepsy Epilepsy limited to females with mental retardation Epilepsy secondary to neurocysticercosis Autoimmune encephalitis

Correct Answer is: Epilepsy limited to females with mental retardation EFMR is a distinctive, under-recognized familial syndrome where girls present with convulsions in infancy, often associated with intellectual impairment and autistic features. The unique inheritance pattern with transmission by males is perplexing. Clinical recognition is straightforward in multiplex families due to the unique inheritance pattern; however, this disorder should be considered in smaller families where females alone have seizures beginning in infancy, particularly in the setting of developmental delay. In single cases, diagnosis will depend on identification of the molecular basis

A 13 year old girl has presented to you with recurrent episodes of being vacant . Recently, she sometimes loses track of her thoughts midsentence. There was one fall off a bicycle that may have been related to one of these events. There seems to be no other associated symptoms , and the episodes may occur up to 10 or more times a day . Her development and health have been normal up to this point apart from a previous head injury at 6 years of age when she fell from a bicycle onto the ground and had a brief loss of consciousness and did not think clearly for the rest of the day , but she required no medical intervention and recovered .You saw this patient urgently as she was due to go to UK for an overseas trip to visit family. Her EEG showed multiple 7-8 second runs of 3-4 Hz generalized discharges of spike. The drug of choice for her would be: Ethosuximide Carbamazebine Phenytoin Lamotrigine Topiramate

Correct Answer is: Ethosuximide The usual age of onset for childhood absence epilepsy (formerly known as petit mal epilepsy) is 4 to 9 years, and for juvenile absence epilepsy is 10 to 15 years. Ethosuximide and sodium valproate are equally effective, but ethosuximide is better tolerated. However, unlike sodium valproate, ethosuximide does not protect against associated generalised tonic-clonic seizures that later occur in about 25% of patients, particularly those with juvenile absence epilepsy. Lamotrigine is less efficacious. Absences may be aggravated by carbamazepine, oxcarbazepine and phenytoin. Treatment is continued until the EEG ceases to show 3 per second spike wave activity and no seizures have occurred for 2 years. However, the persistence of epileptiform abnormality in the EEG is not necessarily a contraindication to the cessation of treatment. In JAE , the consensus is that because of the frequent combination of absences and GTCS, the drug of choice is valproate, which controls all seizures in 70-80% of the patients.

A 3-year-old child with a history of developmental delay is brought to your office for an evaluation. She is accompanied by her maternal grandmother, who has custody of her because of parent neglect. On physical exam, she has mild dysmorphic features with a head circumference at the 5th percentile and height/weight at 10th percentile. She has moderate to severe language delay. She has a smooth philtrum with a thin upper lip. Which of the following is the most likely diagnosis? Autism Down's syndrome Fetal alcohol syndrome Rett's syndrome Trisomy 18

Correct Answer is: Fetal alcohol syndrome The diagnosis of Fetal Alcohol syndrome is based on clinical history of maternal exposure to alcohol and the physical appearance of the child. The diagnostic criteria recommended by the Research Society of Alcoholism Fetal Alcohol study group require : a) presence of prenatal or postnatal growth retardation b) CNS dysfunction such as developmental delay or intellectual impairment; and c) at least two of the following: microcephaly, microphthalmia and short palpebral fissures, or all 3 and hypoplastic philtrum with thin upper lip.

What anticonvulsant is likely to cause life-threatening side effects in the patient described in Question 48? Oxcarbazepine Levitiracetam Clonazepam Valproic acid Ethosuccimide

Correct Answer is: Valproic acid Patients with mitochondrial disorders and other disorders of redox regulation can get irreversible, life-threatening liver disease when they take medications like valproic acid that, in their metabolism, generate reactive oxygen species.

A 14-year-old right handed girl has a sudden onset of a left sided paresis, lasting for 15 minutes, followed by an intense throbbing right sided headache. She previously has had migraine with visual auras. Her mother and aunt have had similar episodes of weakness and headaches as teenagers. On exam in the emergency room, she looks uncomfortable with headache pain, is photophobic, but her weakness has resolved. Which of the following diagnostic tests is most likely to reveal the etiology of this girl's left sided weakness? Carotid ultrasound Diffusion weighted imaging on an Head MRI EEG Genetic testing for the Calcium channel gene CACNA1A Thematic Apperception Test

Correct Answer is: Genetic testing for the Calcium channel gene CACNA1A A new onset of hemiplegia in a teenager is of significant concern. And neuroimaging, including diffusion weighted imaging on an MRI study should be strongly considered. However, in this particular scenario, this young woman most likely has familial hemiplegic migraine. This can be associated with three different genes (CACNA1A, ATP1A2 and SCNA1), with calcium channel mutations (CACNA1A) being commonly detected, particularly if the symptoms first appear before 16 years of age. The hemiplegia may occur before the headache, as is typical for most migrainous auras. A Todd's paralysis, acute stroke, or functional disorder could also be considered in this scenario. Given the strong family and personal history of migraine, this is most likely familial hemiplegic migraine.

A 3-year-old boy is noticed by his father to have difficulty getting up off the floor when playing. He seems to use his hands to "climb" up his own legs. His pediatrician examines him and thinks he feels the liver 2 fingerbreadths below the right costal margin. He obtains liver function tests. The boy's SGOT is 80 U/l; his SGPT is 100 U/l. The pediatrician is concerned, but the studies are only marginally elevated, so he decides to repeat them two weeks later. When the boy comes into the office in two weeks, the pediatrician notices he is waddling from side to side when he walks and sees his calf muscles to be unusually prominent. Once definitive testing is done and returns positive, the pediatrician arranges for his family to talk with a: Hepatologist Nephrologist Sports medicine specialist Geneticist Nutritionist

Correct Answer is: Geneticist This youngster demonstrates a Gower sign, a Trendelenberg waddle, calf hypertrophy, and elevated "liver function tests" almost certainly of muscle origin and released by rhabdomyolysis. He likely has Duchenne muscular dystrophy. His family might benefit from a discussion with a geneticist. Duchenne muscular dystrophy is an X-linked disorder. Knowing this can allow them to make decisions about who else should be tested for carrier or disease status and about having additional children

A 3-year-old boy is brought to the emergency room by his parents with a 3-day history of irritability and 2-day history of unsteadiness in walking and using his arms. He complains of pain in his legs and back and is now unable to walk without assistance. His past medical history is unremarkable except for a 1-week bout of gastroenteritis which resolved last week. He takes no medications. Family history is noncontributory. On examination, he has normal ocular movements and pupillary reactions. He has mild facial weakness. He has mild weakness in both shoulders, but a good handgrip. There is a coarse tremor when reaching for objects. He cannot support his own weight and is unable to sit unsupported. Deep tendon reflexes are absent and plantar responses are flexor. Sensation is intact. Which of the following is the most likely diagnosis? Acute cerebellar ataxia Acute transverse myelitis Botulism Guillain Barre syndrome Polymyositis

Correct Answer is: Guillain Barre syndrome The combination of facial and limb weakness with areflexia of acute onset and rapid progression are most compatible with acute inflammatory demyelinating polyradicuopathy, although a significant subset of these patients have an axonal form rather than demyelinating. Guillain Barre syndrome is an eponym for acute or chronic, axonal or demyelinating inflammatory neuropathy. Presentation can be different in young children than in adults. Up to one third of pediatric cases occur in children less than 3 years of age. In the pediatric group, pain is a common presenting complaint, involving the back or limbs. Ataxia and disturbances of equilibrium rarely observed in adults are present in up to 15% of children. Electrophysiologic studies performed in the first week of the illness most often demonstrate prolonged or absent F-responses (88%), prolonged distal latency (75%), conduction block (58%), and reduction in conduction velocity (50%).

A 3 month old baby has refractory infantile spasms. Prenatally she was found to have a cardiac rhabdomyoma. On examination she has multiple hypomelanotic macules. She has an MRI that reveals frontal cortical tubers . The EEG reveals multifocal discharges; predominantly in the frontal regions. What symptom in her history puts her at risk for developing autism? Having refractory infantile spasms Having a cardiac rhabdomyoma Having hypomelanotic macules Having frontal cortical tubers Having bifrontal discharges

Correct Answer is: Having refractory infantile spasms Current evidence indicates that the likelihood of a child with tuberous sclerosis developing an autism spectrum disorder is greater if the child has a mutation in the TSC2 gene, although autism can and does develop in children with TSC1 mutations. The likelihood is also greater if the child has early-onset infantile spasms that are difficult to control, especially if there is an epileptiform focus in the temporal lobes. The emerging evidence is consistent with the notion that early onset electrophysiological disturbances within the temporal lobes has a deleterious effect on the development and establishment of key social cognitive representations concerned with processing social information, perhaps especially from faces.

An 18 month child presents to your office with his foster parents, who are considering adoption. Perinatal history is unknown except that his new born screens are known to be normal. He is the biologic child of an honors high school students without family history of developmental concerns. His foster parents note that although he is pulling to a stand, he has yet to walk independently. He has "always" used his left hand more than right but is able to use either hand for play with bilateral pincer grasp. He uses only 2 words but will point and prefers his foster parents over strangers. His recent hearing test was normal. He has not had any regression since coming into their home at 2 weeks of age. On exam his weight and height are at the 50% with head size at 10 percentile. He does not have any striking skin findings or dysmorphic features. He babbles jargon and says 'mama', is alert and engaging. His right hand and foot are slightly but definitely smaller than his left. He is strong proximally and distally in all limbs. On passive range of motion he has full range on both sides except for restriction at the end range of his right elbow, wrist and ankle. What testing would be appropriate at this point? A. Head MRI B. Head CT C. Genetic testing with microarray D. A and C

Correct Answer is: Head MRI Language delay and mild right Hemiparesis in the setting of chronic findings (small right side) on exam suggest possible developmental delay in the setting of cerebral palsy CP. CP is a common problem, occurring in about 2 to 2.5 per 1,000 live births. In order to establish that a brain abnormality exists in children with CP that may, in turn, suggest an etiology and prognosis, neuroimaging is recommended with MRI preferred to CT (Level A). Metabolic and genetic studies should not be routinely obtained in the evaluation of the child with CP (Level B). If the clinical history or findings on neuroimaging do not determine a specific structural abnormality or if there are additional and atypical features in the history or clinical examination, metabolic and genetic testing should be considered (Level C). Detection of a brain malformation in a child with CP warrants consideration of an underlying genetic or metabolic etiology.

A 6-month-old girl was born at 36 weeks gestation with microcephaly, petechiae, and hepatosplenomegaly. As a newborn, evaluation revealed intracranial calcifications and sensorineural hearing loss, and the diagnosis of congenital cytomegalovirus (CMV) infection was confirmed. Two weeks ago the patient developed body jerks, with sudden, forceful flexion of the head and neck, torso, and arms, occurring in clusters, more likely upon awakening from sleep. She cries briefly between each jerk, which are 5-8 seconds apart. These clusters can last up to 2 minutes in duration, although some are shorter. Several such clusters are now occurring daily. The EEG pattern that is most likely to be associated with this new symptom is: Burst-suppression background Hypsarrhythmia Photoparoxysmal occipital discharges Spike transients over the right temporal head region 3-Hz generalized spike-wave discharges

Correct Answer is: Hypsarrhythmia Infantile spasms (IS) present with clusters of myoclonic/tonic spasms, with each cluster potentially lasting up to several minutes in duration, and more likely to occur with sleep-wake transitions. They usually begin between 4 and 12 months of age, although older children may also have clusters of epileptic spasms. Many children with IS have pre-existing neurological symptoms. The list of possible etiologies for IS is very diverse, including structural, metabolic, and genetic causes. A small subgroup of patients (10%) has a favorable prognosis, but the outcome for most patients includes at least a mild degree of developmental abnormality and the later appearance of other seizure types. Hypsarrhythmia (or one of its several "modified" forms, or variants) is the EEG pattern most likely to be observed in patients with IS, although the absence of hypsarrhythmia does not exclude the clinical diagnosis

You see a 6 year-old child in the emergency room for gait abnormalities. He is crying, very irritable and unable to ambulate. On examination, he is found to have flaccid paraparesis, impaired movement of the upper extremities, a sensory level at C6 and absent reflexes with no response to Babinski testing. His airway, breathing, and circulation have been assessed and are stable. In addition to obtaining an MRI of the spine, what is the most important and urgent next step in this child's management? Do an LP Insert a urinary catheter Order an MRI of the brain Get an MRI of the spine Perform an EMG/NCS

Correct Answer is: Insert a urinary catheter his child is suffering from a new onset spinal cord syndrome, which may be associated with urinary retention. Although all of the answers above are reasonable steps in evaluation and management of this child, a urinary catheter should be inserted as a first step

You see a 12 year old girl with established cerebral palsy due to complications of premature birth. Despite treatment she continues to experience a poor quality of life due to limits in communication secondary to abnormal tone. They report that stiff posturing of her arms and neck prevents her use of adaptive devices . She has had some improvement in her ability to use her communication board with oral baclofen but experienced sedation at doses where she was most able to accurately activate a multi-choice screen. Botulinum toxin A injections to her biceps has been helpful to reduce pain in the past but the benefit has been fleeting with more recent injections. She did not tolerate side effects of dantrolene and tizanidine. On exam, she is attentive and clearly comprehends the conversation. She is in a wheelchair with head support and attached communication device. She sits quietly until it is her turn to comment, at which point extension of her neck forces her face to the side such that she is unable to maintain gaze on her device. In addition, both arms extend with torsion at the elbow, forceful supination of the wrists and fingers hyperextended or flexed such that she is unable to accurately reach the device choices. Later in the exam she is starting to nap on the exam table, you are easily able to obtain near full passive range of motion of her upper limbs despite velocity dependent spastic catch. She has significant limited passive extension of her knees and her parents confirm she does not ambulate . Which surgical intervention(s) would be reasonable to consider: Selective dorsal rhizotomy Intrathecal baclofen Tendon transfer forearm/wrist Phenol block biceps

Correct Answer is: Intrathecal baclofen This young lady has identified difficulty with communication /use of adaptive devices as the cause of her poor quality of life. Her exam suggests she is impaired by her dystonia. Although she has spasticity by history and exam, she does not have upper limb contractures. Intrathecal baclofen is an appropriate intervention to consider for in moderate or severe dystonia, which prevents function. It is important for patients to have a thorough multi-disciplinary assessment and appropriate trial prior to committing to any surgical procedure, including the baclofen pump. Selective dorsal rhizotomy SDR is used most often for children with spastic diplegia limiting ambulation and specifically targets spasticity. Her status using wheelchair for mobility , contractures and main disability affecting her upper limbs makes her a poor SDR candidate. Although tenotomy/tendon transfer and phenol blocks may reduce effects of spasticity and dystonia to relieve pain and improve focal dysfunction, they will not fully address her generalized dystonia which is her primary concern. The risk that these interventions could reduce the strength of a spastic muscle group might not be acceptable for unclear benefit.

A 3-year-old boy with precursor B cell acute lymphoblastic leukemia is being seen after his nurse notes that he is no longer using his left arm. He was originally admitted for induction chemotherapy. Thus far, he has received intrathecal cytarabine; systemic methylprednisolone, doxorubicin, vincristine, and asparaginase; and intrathecal triple therapy with cytarabine, methotrexate, and hydrocortisone. His last intrathecal treatment was ten days ago, and he tolerated the procedure well. On neurologic examination, he has slurred speech, left lower facial droop, and left arm paralysis. Diffusion-weighted MR imaging shows bilateral areas of restricted diffusion in the cerebral white matter, most prominently within the right temporal lobe. His symptoms gradually resolve over the following week, and repeat MR imaging one month later shows complete resolution of the diffusion abnormalities. Which of the following medications is most likely to have caused this patient's symptoms? Doxorubicin Intrathecal hydrocortisone Intrathecal methotrexate Methylprednisolone Vincristine

Correct Answer is: Intrathecal methotrexate Childhood leukemia is associated with a number of central nervous system complications, some secondary to the disease itself and others secondary to its treatment. Leukemia can cause neurologic dysfunction through direct invasion of the nervous system, which generally occurs through leptomeningeal metastasis with dissemination of leukemic cells throughout the subarachnoid space. It can also cause dysfunction indirectly, often through disturbances of coagulation and blood flow (e.g. disseminated intravascular coagulation, hyperviscosity/leukostasis, venous sinus thrombosis). Finally, the CNS is a common site of relapse, as seen in 3-8% of children with either ALL or AML. The treatments for childhood leukemia—systemic/intrathecal chemotherapy, cranial irradiation, and bone marrow transplantation—can also cause neurologic dysfunction. The side effects of chemotherapy vary based on the specific agents used. Intrathecal and high-dose intravenous methotrexate have been shown to cause immediate, acute to subacute, and chronic neurologic syndromes. The acute to subacute syndrome described in this case typically occurs within days to several weeks of treatment and is characterized by a variety of neurologic signs, including seizures, mental status changes, and focal neurologic deficits (e.g. aphasia, anesthesia, paresis, ataxia, tremor, blurred vision). Brain imaging reveals leukoencephalopathy, but the lesions are usually reversible. Intrathecal chemotherapy—whether with methotrexate or other agents—can also be associated with complications such as post-dural puncture headache, CNS hemorrhage, spinal cord dysfunction, chemical meningitis, and neuropsychological dysfunction. Cranial irradiation is a very effective form of CNSdirected treatment, but attempts have been made to eliminate or reduce its use given its substantial rates of secondary neoplasms, endocrine diseases, growth impairment, neurocognitive dysfunction, and other neurotoxic effects. Patients who undergo bone marrow transplantion are at risk for seizures, which occur in about 5% of patients treated with cyclosporine; CNS infections; and graft-versus host disease.

A 5-year-old boy has generalized tonic-clonic seizures and recently, he has begun to fall to the ground limp. His Kindergarten teacher has been complaining that he sometimes "blanks out" in class. She thinks his overall cognitive function, particularly his language, has been deteriorating. His EEG shows 1-to-2 Hz spike-and-wave discharges that are almost continuous during sleep. Which of the following is the most likely diagnosis? Juvenile myoclonic epilepsy Lennox-Gastaut syndrome Mitochondrial encephalopathy Landau-Kleffner syndrome Childhood absence epilepy

Correct Answer is: Landau-Kleffner syndrome Landau-Kleffner Syndrome (LKS) is defined as an epileptic encephalopathy in which deterioration of cognitive, sensory, and/or motor functions results from epileptic activity. LKS is associated with continuous or almost continuous spike-and-wave discharges during slow wave sleep. As soon as the patient falls asleep, continuous and diffuse slow spikes and waves appear, mainly at 1.5-2.5 Hz, persisting through all the slow-sleep stages. Semiology of the seizures observed in LKS can vary, but includes partial complex (with focal motor and atypical absence symptoms), partial clonic, generalized tonic-clonic, and atonic seizures, By convention LKS is not related to organic brain lesions and occurs in previously normal children who have already developed age-appropriate speech. However, there are reports of "clinically defined" LKS in patients with a congenital or acquired brain lesion and in children who demonstrated substantial pre-existing anomalies of language development prior to their language regression.

A 17 year old patient has transferred her care to you. During the appointment she mentioned her reproductive health. She is not pregnant but she would like to know how the medications that are prescribed can affect a fetus. She takes phenytoin for epilepsy. Her psychiatrist has prescribed valproic acid but has added lithium for severe bipolar disorder. She also states that her dermatologist is considering treating her severe acne with isotretinoin. (Accutane). In utero exposure of which medication does not increase the risk of intellectual disability? Phenytoin Valproic acid Isotretinoin Lithium All of the medications increase the risk of intellectual disability in the fetus

Correct Answer is: Lithium The fetal hydantoin syndrome consisting of facial dysmorphism, cleft lip and palate, cardiac defects, digital hypoplasia, and nail dysplasia was initially ascribed to the use of phenytoin during pregnancy. Phenytoin has also been shown to decrease IQ points. Valproic acid has been shown to increase the risk of neural tube defects, Autism and intellectual disability. Isotretinoin has been shown to cause microcephaly, hydrocephalus and intellectual disability. Lithium has been shown to cause Ebstein anomaly but not an intellectual disability.

A 22-month-old boy is referred for evaluation of severe expressive and receptive language delay. He is unable to point. He has poor eye contact and pulls his parents around the house to indicate his needs. He likes to line up his toys and gets upset when they are taken out of order. He also flaps his hands whenever he is excited. On physical exam, he has two hypopigmented macules on his trunk. Which of the following should be done next? Electroencephalogram Fragile X testing High resolution chromosomes Modified Checklist for Autism in Toddlers Skull X-Rays

Correct Answer is: Modified Checklist for Autism in Toddlers The child in this vignette has classical symptoms of autism. Autism is a neurodevelopmental disorder characterized by impaired social communication, delayed expressive and receptive language and stereotypic behavior. The language disorder is characterized by delay in the development of spoken language or difficulties sustaining speech. This is also a social disorder with marked impairment in the use of nonverbal behaviors like gestures. There are restricted patterns of behavior like finger flapping, twisting or complex body movements. There may be absence of imaginative play. The cause for most cases of autism is elusive. However, there are genetic disorders that are associated with autism spectrum disorders with Rett's syndrome, Fragile X and tuberous sclerosis complex. In this vignette, the patient has classical symptoms of autism. Early identification of this disorder will allow specific interventions to improve his outcome. The Modified checklist for autism in toddlers is an easy tool to use for the pediatrician or primary provider. However, it is recommended that a multidisciplinary team that specializes in children with these disorders evaluate this child. The most common measures used include the Childhood Autism rating scales (trained observer), Gilliam Autism rating scale (parents report). The gold standard is the Autism Diagnostic Observation Schedule (ADDOS) or the Autism Diagnostic InterviewRevised (AID-R) . The hypopigmented spots raise concern of tuberous sclerosis, so an MRI might also need to be performed.

A 6-year-old, Caucasian boy presents to the office for evaluation of developmental delay. He was born full-term following an unremarkable pregnancy but remained in the hospital for 2 days secondary to problems with feeding and jaundice. Birth weight was 7 lbs. 8 oz. (25-50%ile) and length was 22 in. (>95%ile). On examination, he is noted to be tall and macrocephalic (both > 97 percentile) with mildly dysmorphic facial features, including a high, broad forehead, down-slanting palpebral fissures, and a pointed chin. His hands and feet are enlarged. His neurologic examination is non-focal. There are no similarly affected individuals in the family. Which genetic test is most likely to yield a positive result? Chromosome microarray FISH for chromosome 22q11 deletion Karyotype Methylation analysis of chromosome 15q Nuclear receptor SET domain containing protein-1 (NSD1 testing)

Correct Answer is: Nuclear receptor SET domain containing protein-1 (NSD1 testing) Sotos syndrome, also known as cerebral gigantism, is a genetic condition that affects 1:10,000 to 1:50,000 individuals. The syndrome is characterized by prenatal and postnatal overgrowth, macrocephaly, and characteristic facial features—specifically a high anterior hair line; frontotemporal hair sparsity, a long, thin face; frontal bossing; downslanting palpebral fissures; and a prominent mandible. Advanced bone age is common but is not a universal finding. Neurologic hallmarks include hypotonia and global developmental delay, with most affected adults exhibiting mild intellectual disability. Sotos syndrome can be caused by either intragenic truncating mutations or microdeletions that result in haploinsufficiency of the nuclear receptor binding SET domain protein 1 (NSD1), located on chromosome 5q35.3. Intragenic mutations account for more than 80% of cases in European-origin white patients, wheres microdeletions occur with a higher incidence in the Japanese population. The majority of cases are sporadic, though autosomal dominant pedigrees have been reported.

A 5-year-old-boy has been in preschool for two years. The teachers have counseled the mother that he is slower compared to the rest of his classmates and that he may need an extra year of preschool. The family is as asking you to determine if he is ready for Kindergarten this coming school year. His physical and neurological exam are normal. Which of the following is the most appropriate next step in management? Continue to observe Electroencephalogram Magnetic resonance imaging of the Head Your Answer Obtain standardized neurocognitive testing Prescribe an atypical antipsychotic medication

Correct Answer is: Obtain standardized neurocognitive testing In this vignette, the child with a history of developmental delay is at risk for learning disabilities or mental retardation. Mental retardation is defined as "significantly sub-average general intellectual functioning with an intelligence quotient score (IQ) of < 70, existing concurrently with deficits in adaptive behavior and manifested during the developmental period, which adversely affects a child's educational performance". Cognitive testing employed including the Stanford Binet Intelligence Scales-Fifth Edition (SB5), Wechsler Intelligence Scales for children, Wechsler Individual Achievement tests and, Mullen scales of early learning and Wide Range Test for Achievement. However, there also needs to be concurrent difficulties in adaptive behavior. The common questionnaires for an adaptive behavior include Vineland scales. It is important to identify these children at risk for these disorders to guide the school, clinicians and families with regards to his educational needs in school. Guidelines from the American Academy of Neurology state that assessment of the child with developmental delay might also include MRI and genetic testing.

The parents of the child described in Question 35 (failed to find Q35) ask for information regarding risk of recurrence and possible evolution to MS. Which factors are useful in considering risk stratification for recurrence in this population? Family history and second hand smoke exposure Second hand smoke exposure and patient age Patient age and presence of brain lesions Presence of brain lesions and developmental history Developmental history and family history

Correct Answer is: Patient age and presence of brain lesions Risk stratification using age and presence of brain lesions has been evaluated in a national prospective cohort study of Canadian children with new onset demyelination (). Younger children with no brain lesions are at very low risk for development of MS.

You have admitted your 6 year old patient with Autism to the hospital due to gait abnormality. His parents state that he has refused to walk for several days. He also is more irritable. His parents state that he has not been eating very much. In fact, his parents state for the past several months his food aversion has worsened; he only eats chicken nuggets and drinks only water. On examination you note he is very thin. His gums start bleeding after you look into his mouth. You also note bruising on lower legs and his buttocks. His right leg is tender to palpation. You order a CBC which revealed anemia. A BMP was negative. An XRAY of his lower legs reveal ground glass osteopenia. A deficiency in which Vitamin should be considered in this child with restricted intake of foods? Vitamin A Vitamin B .Vitamin C Vitamin D Vitamin E

Correct Answer is: Vitamin C The child above has scurvy. It is an ancient disease that has become more common in developmentally delayed children with restricted diets. Scurvy today is usually found among the elderly, the poor, alcoholics, and those restricted to macrobiotic diets or other diets that contain <10 mg/day of vitamin C. Manifestations of scurvy are largely related to underlying small hemorrhages, abnormal mitochondrial metabolism, and abnormal collagen synthesis and may manifest as bleeding gums and hemarthroses, arthralgias, malaise, and weakness. Review of the literature reveals 23 case reports of scurvy in children with behaviorally restricted diets , including 2 children with autism and another with an unspecified behavioral problem, 1 with trisomy 21, 1 with static encephalopathy after pertussis-associated encephalitis, 2 with cerebral palsy, 4 with mental retardation, and 5 with global developmental delay; 7 otherwise reportedly normal children, including 1 morbidly obese teenager and another with a pathologic grief reaction, had scurvy in association with unusual diets and isolated foodavoidant behaviors.

A 7-year-old girl was riding her tricycle down the sidewalk when an adolescent on a skateboard sped in the opposite direction and knocked her off the seat. The child's left leg hit the sidewalk, scraping the skin and bruising it, and the tricycle toppled on top of her, but did not cause serious injury. She cried briefly and walked her tricycle back to her house, just a few yards down the street. Two weeks later, she began refusing to walk on that leg. Her parents noticed that the leg would become bright red, hot, and swollen from the knee down and stay that way for hours at a time. The child might benefit from: Bed rest and pampering. Antibiotics and topical vasoconstrictors. . Physical therapy Oxycodone and prednisone. Casting and avoidance of weight bearing.

Correct Answer is: Physical therapy Complex regional pain syndrome (formerly known as reflex sympathetic dystrophy) is well-described in children but poorly understood in any age group. Multifactorial etiology and predisposition are suspected and therapy is almost always multidisciplinary, involving graded physical therapy, individual and family psychotherapy, agents aimed at decreasing neurogenic pain, and agents that affect the autonomic nervous system.

A 17-year-old girl is being seen for chronic headaches, problems falling asleep, upsetting dreams, and decreased school performance, all of which have been worsening over the past several months. Her mother states that the teenager has been losing interest in school activities and in socializing with her friends, and has been increasingly irritable, moody, and angry. During the interview, you find out that her father was killed in a car accident six months ago and she broke up with her boyfriend after numerous verbal fights three months ago. She has been afraid to drive a car and has had nightmares about her father's death. Her headaches are "all over my head" and constant in nature. Their intensity does not vary over the course of the day or from day to day. She has a normal neurological and general physical examination. The most likely diagnosis for this girl's difficulties is: Depression Post-traumatic stress disorder Bipolar disorder Insomnia Learning disability

Correct Answer is: Post-traumatic stress disorder All children and adults can experience stressful events that lead to both emotional and physical consequences. While reactions to stressful events are usually brief and associated with full recovery, exposure to catastrophic or repeated traumatic events can lead to unconscious mental dissociation, an emotional numbing to deaden the pain and trauma. It is important to realize that, since younger children have limited verbal skills and non-specific ways to react to stress, they may not always meet DSM criteria for post-traumatic stress disorder. Children generally exhibit more emotional lability after severe stress than do adult

The 6 year old child described in Question 35 (Failed to find Q35) receives an MRI of the brain and spine. The brain MRI is normal, but the spine MRI is not. There is a single, irregularly shaped area at C6 of hyperintense signal seen on T2 and seen best on FLARE sequences. Lumbar puncture shows 15 WBC, 0 RBC and a normal protein and glucose. What first line therapy may be helpful in the management of this case? Pulse corticosteroids (20-40 mg/kg/d) IVIG Interferon beta Cyclophosphmide Rituxima

Correct Answer is: Pulse corticosteroids (20-40 mg/kg/d) This child has acute transverse myelitis with a demyelinating lesion on MRI. The standard treatment is pulse high-dose IV steroids begun as soon as possible after onset of symptoms. The urgency of need for treatment is underscored by this child's urinary retention.

A 1-week-old girl is seen for unusual jerking movements during sleep. She was born at term gestation, with an uncomplicated pregnancy and delivery. She was discharged home at one day of age without recognized problems. She seems otherwise healthy and alert to her parents. However, for the past 3 days, shortly after she goes to sleep she develops repetitive, arrhythmical jerks of both arms, sometimes stronger on one side of the body or the other. These usually last for 2 minutes then disappear. They do not awaken her. These can sometimes be provoked by touching her or picking her up while asleep, but they disappear if she is awakened. There is no family history for this type of symptom. She is examined while awake, with normal findings for her age. Her routine EEG is normal, while awake and during sleep, although no clinical events occurred during the EEG study. What is the single best treatment choice for this patient? Adrenocorticotropin hormone (ACTH) injections daily Diazepam rectal gel given at the times of the events Phenobarbital given orally Pyridoxine intravenous trial Re-assurance and continued observation

Correct Answer is: Re-assurance and continued observation Benign Neonatal Sleep Myoclonus is probably under-diagnosed relative to its true incidence in the population. It occurs exclusively during sleep. Myoclonus may occur in any sleep stage, but they are reportedly less common during active sleep. They most commonly begin during the first two weeks of life, and disappear within 3-6 months. The jerks are often bilateral, affect the arms more than the legs or the face, and are irregularly repetitive. They last less than a minute in most patients, but can be prolonged, lasting up to 30 minutes or even an hour if the child is not awakened. The EEG is normal between and during the spells. If the diagnosis is correct, a normal neurological and developmental outcome is expected. Anti-epilepsy drugs (AEDs) are not advised, and have been reported to worsen some patients with this condition

A 3 day old boy , was being observed in Special Care Nursery for respiratory distress due to likely transient tachypnea of the newborn and was to be discharged back to his mother. The nurse noted rhythmic jerking movements of his right arm . He was born at term gestation, with an otherwise uncomplicated pregnancy and delivery. The jerking lasted for 1 minute and subsided. The infant slept through the event. Retrospectively, the nurse recalled repetitive jerks of his left arm on her last shift, but when she tried to examine him, they disappeared, and he woke up . He is otherwise well. There is no similar family history. His head circumference, length, and weight are normal . The neonatologist did an EEG which was normal. What treatment would you advise? Pyridoxine and folic acid intravenous trial Advice upon discharge to give diazepam rectal gel given at the times of the events if they are prolonged Phenobarbital load and oral maintenance Reassurance and routine follow up Adrenocorticotropin hormone (ACTH) injections daily

Correct Answer is: Reassurance and routine follow up This child has benign sleep myoclonus. Benign sleep myoclonus refers to often multifocal, migrating limb movements during drowsy and sleep states only, which stop on arousal and may worsen with light touch or sound insufficient to awaken the subject. It can frequently be observed beyond the neonatal period. Benign sleep myoclonus is observed most commonly in the arms, and the movements can occur in rapid succession, causing concern to caregivers. The movements most commonly begin during the first two weeks of life, and disappear within 3-6 months. If the diagnosis is correct, a normal neurological and developmental outcome is expected. Anti-epileptic drugs are not advised, and have been reported to worsen some patients with this condition.

A 4-month-old boy from China is adopted by a couple in the U.S. Soon after they bring him home, he begins having episodes of inconsolable screaming, arching of his back and neck, and apparent lack of awareness of his surroundings. On exam, he has no deep tendon reflexes but has paradoxically increased tone both axially and appendicularly. An MRI scan of his head would be expected to demonstrate abnormalities of the: Skull Grey matter White matter Venous sinuses Basal ganglia

Correct Answer is: White matter This description is most consistent with a diagnosis of Krabbe disease, a leukodystrophy. The fact that he was not born in the U.S. explains the absence of the newborn testing that might have made this family aware of the diagnosis before the adoption

An 8-year-old boy is eating dinner with his family and tells his parents and siblings about a cool "superpower" he has. "I can sit in my seat in class and make the teacher get bigger and smaller and bigger and smaller just by really concentrating hard." His older brother rolls his eyes and his younger sister starts to giggle. His parents are concerned and the next day after school, his mother takes him to an optometrist to have his eyes checked. His eyes and vision are normal, so she brings him to the pediatrician during walk-in hours. The pediatrician finds that the family history is positive for migraine in the child's maternal grandmother and aunt. The child's father was a sleepwalker as a child. The child's mother has to sit in the front seat of the car when she is a passenger, lest she vomit. The pediatrician should: Order an MRI scan for the child. Tell the child to stop scaring his parents. Reassure the family by telling them what the episodes likely are. Have the mother bring her other children in for neurological evaluation. Refer the child to a psychiatrist or psychologist.

Correct Answer is: Reassure the family by telling them what the episodes likely are. Alice in Wonderland syndrome is a migraine variant that is unaccompanied by pain, nausea, or photo-/phonophobia and that serves as a marker for the migraine-susceptible genotype. It generally occurs in children who have a family history of migraine and associated phenomena and who are otherwise neurologically normal

John is a 17 year old boy who presented to the emergency department with an episode of convulsive status epilepticus for 45 min. There was no apparent aura, and his head and eyes deviated to the left. This episode was precipitated by a urinary tract infection and fever to 100⁰F. John's last such episode was 1 year ago and lasted for 30 minutes. He has had and continues to have many other types of seizures, including focal dyscognitive seizures, myoclonic seizures, absence seizures, and brief nocturnal tonic clonic seizures. John's seizure history started with febrile status epilepticus at 6 months. John had normal development until 18 months of age, at which time his development regressed. He now has severe intellectual disability. His current daily medications are valproate, clobazam. and topiramate. John most likely has which genetic mutation? CACNA1H CLCN2 PCDH19 SCN1A STXBP1

Correct Answer is: SCN1A John has Dravet's syndrome, first described by Charlotte Dravet in 1978. Severe myoclonic epilepsy in infants (SMEI), also known as Dravet syndrome (Dravet 2005), is one of the most refractory forms of epilepsy Seizures often present with multiple febrile seizures or febrile status epilepticus . Seizures could include various phenotypes including myoclonic, atypical absences, atonic seizures and focal seizures, which worsen with fever Development may be normal prior to onset. We later see regression in language and motor skills. De novo mutations in the sodium channel SCN1A are a major cause of this syndrome (Fujiwara 2003) and have been found in 40% to 70% of patients with typical SMEI. Recently identified responsible genes include SCN1A, CDKL5 PCDH19, and GABRG2

A 6-week-old boy is brought to the pediatrician for his first well-child visit. He seemed normal to his parents when they brought him home. But by 4 weeks of age, he seemed to have become floppier. Over the past two weeks, he seems to be moving his arms and legs a lot less and he can no longer hold his head up. In the past week, he seems to his mother to get fatigued while nursing. On examination, he is alert and awake. He has a very expressive face for one so young! He does not lift his arms or legs against gravity and has absent deep tendon reflexes. He is likely to have an abnormality of the gene: MYCN SMN1 NGF1 abcd1 BCL2

Correct Answer is: SMN1 This child's presentation is characteristic of Type 1 Spinal Muscular Atrophy or Werndig-Hoffman disease. This disease is most commonly related to a mutation of the telomeric copy of the SMN1 gene, which codes for the survival of motor neuron 1 protein.

A 9-month-old girl is evaluated for extreme irritability, failure to thrive, vomiting, and possible seizures. She was born at term, birth weight 3.8 kg, with sudden fetal distress during labor and associated with uterine rupture and placental abruption. She was delivered by emergency caesarean section. She had significant hypoxic-ischemic encephalopathy with severe metabolic acidosis and neonatal depression. There were no seizures. Subsequently she has developed very slowly, with signs of increased tone, and microcephaly. Over the past two months she has been vomiting several times daily, with increased irritability, and poor weight gain. Over the past two weeks she has also developed peculiar spells, with tonic extension of the neck and back, while turning the head forcefully to her left side and crying as if in pain. These events may last for 30 minutes. An EEG showed a mild degree of generalized slowing, but no epileptiform abnormality. Which of the following is the most likely explanation for these spells? Frontal lobe seizures Infantile spasms Pallid breath-holding spells Sandifer syndrome Increased intracranial pressure

Correct Answer is: Sandifer syndrome Sandifer syndrome is related to gastro-esophageal reflux disease (GERD) and presumably results from irritability and discomfort associated with reflux esophagitis. The symptoms consist of episodic dystonic posturing with torticollis, usually with crying or other signs of distress. Otherwise normal children with severe untreated reflux may develop Sandifer syndrome, but it is much more likely to occur in children with pre-existing neurological abnormalities, who are at increased risk for GERD. Many of these children have epilepsy, and the events of Sandifer syndrome may be mistaken for epileptic seizure events. Identifying the correct diagnosis is important, as anti-reflux therapy may be dramatically effective.

A 3-year-old girl from Guatemala is referred to you for progressive refusal to walk and absent deep tendon reflexes on her pediatrician's examination. You notice that she keeps her hips and knees flexed, even when you support her from both axillae to hold her up in vertical suspension. When you try to put the soles of her feet onto the examining table, she screams as if she is in pain. Rubbing the bottoms of her feet with your hands elicits the same response. When you lay her flat on the table and passively extend her legs, you notice they are bowed. Her parents state that her diet consists of juice and chicken nuggets. You think you know what is wrong and test your hypothesis by ordering: An MRI of the brain A serum lactate Serum calcium, phosphate, alkaline phosphatase, and vitamin D A Spanish language psychiatric evaluation. A somatosensory evoked potential study.

Correct Answer is: Serum calcium, phosphate, alkaline phosphatase, and vitamin D Longstanding vitamin D deficiency in toddlers causes a painful neuropathy, bowing of the legs, a low serum calcium, a low serum phosphate, and a high serum alkaline phosphatase. The neuropathy may be evident on an EMG/NCV study, but this study will not allow determination of the etiology of the neuropathy. The other studies would be expected to be unhelpful in this child.

A 2-month-old baby boy is brought to the physician by his parents because he is floppy compared to their first child. He does not lift his head or roll over and he perspires profusely on his head and face when feeding. He is alert and smiles appropriately but has scant limb movements and a weak cry. On examination the child appears hypotonic and weak. He is arreflexic and the muscles are difficult to palpate. A specific diagnosis is most likely to be established by which of the following studies? Serum CK Blood spot for alpha 1-4 glucosidase deficiency. Determination of CTG repeat number in the PK gene. Survival motor neuron gene mutation analysis Electromyography.

Correct Answer is: Survival motor neuron gene mutation analysis Perspiration when feeding is an indication of the work the infant is doing to breath and eat at the same time. The expressive face would be expected in SMA at this stage but not in myotonic dystrophy (type 1 associated with PK gene abnormality). Although Pompe's disease would not affect the facial expression at this age, the fact that the muscles are not firm and easily palpated would be unexpected. Though the serum CK, EMG, MRI and muscle biopsy are reasonable tests to consider, the most common cause of hypotonia and weakness in this age group, particularly given the alert and expressive face and flabby muscles would SMA-1.

A three year old boy is brought to you for a neurological evaluation. During the office visit you note that he does not have joint attention, makes minimal eye contact, has echolalia and repetitively spins instead of playing with the toys in the room. You are concerned that he has an Autism Spectrum Disorder. In obtaining the pregnancy history, mom states that she had taken valproic acid and topiramate for severe migraines during the pregnancy. The valproic acid was added after the first trimester. She had also taken folic acid. She would like to know if any of the medications could have contributed to the child's symptoms. Your response would be: Only thalidomide has been shown to increase the risk of Autism Taking valproic acid only increases the risk of spinal tube defects Taking topiramate does not cause any birth defects The risk of autism increased 3-fold for children born to mothers who took VPA during pregnancy The risk of autism is similar to spina bifida; it only increases during first trimester exposure

Correct Answer is: The risk of autism increased 3-fold for children born to mothers who took VPA during pregnancy In utero exposures to Thalidomide and Valproic Acid have been shown to increase the risk of Autism. It is well known that Valproic Acid exposure can increase the risk of neural tube defects. The study by Christensen et al and the commentary by Cynthia Harden revealed there "were an association of in utero VPA exposure and the risk of autism determined from a population-based study utilizing Danish national medical databases. Children born between 1996 and 2006 were followed for the documented occurrence of autism up to 14 years of age, with a mean follow-up of 9 years. The results were that for children born to mothers who took VPA during pregnancy, the risk of childhood autism increased 3-fold and the risk of autism spectrum disorder (ASD) increased 5-fold over the general population risk. The absolute risk to VPA-exposed children was less than 5%, although the upper 95% confidence interval (CI) limit for ASD was 7.49%. There was no contribution of maternal epilepsy to the risk of VPA use. In contrast to the structural teratogenesis with VPA, which manifests with hypospadias and spina bifida and occurs with first trimester exposure, the autism risk was present in this study with VPA exposure starting at any trimester during pregnancy." In regard to Thalidomide exposure, there was a 50 fold higher rate of Autism in those with thalidomide embryopathy than in the general population. In regard to Topiramate, data from the North American Antiepileptic Drug (AED) Pregnancy Registry indicate an increased risk of oral clefts in infants exposed to topiramate during the first trimester of pregnancy. Infants exposed to topiramate as a single therapy experienced a 1.4 percent prevalence of oral clefts, compared with a prevalence of 0.38 percent - 0.55 percent in infants exposed to other antiepileptic drugs.

A 10-year-old girl with Down syndrome presents to your office. Her family is concerned that she has recently manifested regression of her self-help skills. She has been irritable lately, and this is also quite unusual for her. She had been doing well in special education classes, but recently, her grades have deteriorated. She has a flat facial profile and almond-shaped palpebral fissures. Her neurological examination reveals mild hypotonia with 1+ deep tendon reflexes throughout. Her knee jerk takes a fraction of a second longer than usual to recoil, but her Babinski reflex is absent. The most appropriate next step in her evaluation is: CT of the brain Thyroid function tests Cervical spine films Electromyography Spinal fluid for neurotransmitters

Correct Answer is: Thyroid function tests Down syndrome is a chromosomal disorder that results from non-dysjunction of chromosome 21 during gamete formation. Three copies of chromosome 21 are present. Children with Down syndrome often have complex systemic abnormalities at birth, including congenital heart disease and duodenal atresia. Atlanto-occipital ligamentous laxity is common, and hypothyroidism can be congenital or acquired and persistent or transient. The constellation of symptoms and signs exhibited by this child suggest hypothyroidism.

A 16-year-old girl has had migraine headaches since early childhood. She has always taken ibuprofen for them. For many years, a single dose of ibuprofen worked for each headache, and the headaches came only once or twice per month. Now, the headaches come most days of each week, and she is taking ibuprofen 2 or 3 times a day almost every day. Over the past four weeks, the headaches have become more severe and less responsive to ibuprofen. They have not changed in character, and her mother gets "the same headache" and takes something every night at bedtime to prevent them from coming. Her physical examination is normal with the exception of a BMI of 30. You tell her she has to stop the ibuprofen "cold turkey" and give her a triptan to use no more than twice a week. You might best decide to also give her daily preventive treatment with: Naproxen sodium Valproic acid Topiramate Botulinum toxin Verapamil

Correct Answer is: Topiramate This adolescent girl has a past medical history remarkable for migraine. Her migraines have become more frequent during adolescence, a phenomenon that is quite common, especially in young women. Because of this, she begins taking ibuprofen, an NSAID, multiple times a day almost every day. She has therefore developed medication overuse headaches. Interestingly, NSAIDs appear to be efficacious and even protective in individuals with infrequent headaches (as this youngster was in early childhood), but likely to trigger medication overuse headaches in those with frequent or chronic headaches (as she became recently). Treatment involves discontinuation of NSAID, symptomatic use of triptans no more than twice a day for no more than 2 days in any week, and institution of a daily preventive medication for migraine. Of those listed, verapamil is not particularly efficacious and valproic acid is likely to cause weight gain and, possibly, ovarian cysts in this already overweight youngster. On the other hand, topiramate is both efficacious and likely to cause weight loss, so it is a good firstline preventive drug for this adolescent.

A 5-year-old boy develops an in-toeing of his right foot with walking. Within the next year, this dystonic posturing generalizes to involve his right arm, and the left side of his body as well. His parents report no cognitive problems, and he has had no seizures. There is no diurnal variation. On exam, he has a generalized dytsonia involving all extremeties. His MRI scan is normal. This boy most likely has a genetic defect involving which of the following proteins (genes)? Epsilon-sarcoglycan (SCGE) GTP cyclohydrolase 1 (GTPCH 1) Pantothenate kinase (PANK2) Thyroid transcription factor 1 (TITF1) Torsion A (DYT1)

Correct Answer is: Torsion A (DYT1) DYT1 dystonia (also referred to as torsion dystonia or dystonia musulorum deformans) typically presents with dystonic posturing of a foot, leg, or arm. Dystonia is usually first apparent with specific actions; e.g., writing or walking. Over time, the contractions frequently become evident with less specific actions and spread to other body regions. Myoclonus-dystonia (M-D) is a movement disorder characterized by a combination of myoclonus and dystonia. Additional psychiatric features can include depression, anxiety, and obsessive-compulsive disorder (OCD). It is associated with mutations in the epsilon-sarcoglycan gene. GTP cyclohydrolase 1-deficient dopa-responsive dystonia (GTPCH1) is characterized by a dramatic response to low doses of levodopa. This disorder typically presents with gait disturbance caused by foot dystonia, later development of parkinsonism, and diurnal fluctuation of symptoms. Pantothenate kinase-associated neurodegeneration (PKAN) is a form of neurodegeneration with brain iron accumulation and was formerly called Hallervorden-Spatz syndrome. PKAN is characterized by progressive dystonia and basal ganglia iron deposition that results in an "eye of the tiger" sign on MRI. Benign hereditary chorea is an autosomal dominant disorder with nonprogressive chorea. It is caused by mutations in the thyroid transcription factor (TITF-1) gene.

Which of the following syndromes is not typically associated with an intellectual disability? Fragile X syndrome Rett Syndrome Treacher Collins Syndrome de Morsier's syndrome Down Syndrome

Correct Answer is: Treacher Collins Syndrome Down syndrome is the leading inherited cause of intellectual disability. Fragile X is the leading inherited cause of intellectual disability in boys. Rett syndrome is a common X-linked cause of syndromic intellectual disability. Intellectual disability is a common finding in de Morsier's syndrome (septo-optic dysplasia) and can occur in 50% of the patients. Treacher Collins Syndrome is a syndrome involving the development of the facial bones. Those affected have normal intelligence

Angelman syndrome (AS) is a neurodevelopmental disorder characterized by severe mental retardation, absent speech, ataxia, sociable affect, and dysmorphic facial features. If genetic testing for the maternal deletion in the 15q11.2-q13 critical region was negative, what genetic test should be considered next? FMR UBE3A ARX LIS TSC

Correct Answer is: UBE3A Angelman syndrome is a neurodevelopmental disorder characterized by mental retardation, movement or balance disorder, typical abnormal behaviors, and severe limitations in speech and language. Most cases are caused by absence of a maternal contribution to the imprinted region on chromosome 15q11-q13. Approximately 70% of AS cases result from de novo maternal deletions involving chromosome 15q11.2-q13; approximately 2% result from paternal uniparental disomy of 15q11.2-q13; and 2 to 3% result from imprinting defects. A subset of the remaining 25% is caused by mutations in the gene encoding the ubiquitin-protein ligase E3A gene (UBE3A). Several patients with a clinical diagnosis of Angelman-like syndrome have been reported with mutations in the MECP2 gene.

An eleven month old girl presented to the emergency in status epililepticus . She failed to respond to 2 doses of lorazepam, was loaded with phenobarbitone and phenytoin , and underwent rapid sequence intubation and is currently also on Levetiracetam , Lacosamide and Midazolam infusion of 3 mcg/kg/min, On a review of history she had a global developmental delay and deranged liver enzymes . She still has ongoing seizures. Her EEG has slowing suggestive of severe encephalopathy. Her two older siblings are also globally delayed and her mother has had 3 previous miscarriages. On the basis of the above history it would be most important to: avoid carbamazepine, in view of suspected Dravet's syndrome avoid valproate, in view of Alpers' syndrome avoid phenytoin, in view of Doose syndrome try all of the above in an effort to treat status epilepticus avoid topiramate as it causes metabolic acidosis

Correct Answer is: avoid valproate, in view of Alpers' syndrome The above history is suggestive of a mitochondrial disorder, of the above choices it would be most important to avoid valproate . The above history is not typically suggestive of Dravet's or Doose syndrome. Rapid onset of liver failure is described when valproic acid and sodium divalproate (divalproex) have been used to treat seizures. This child was tested and was positive for POLG mutation and was diagnosed as Alpers disease/Alpers-Huttenlocher syndrome (AHS). AHS, one of the most severe phenotypic manifestations in the spectrum of POLG-related disorders, is characterized by a progressive and ultimately severe encephalopathy with intractable epilepsy, neuropathy, and hepatic failure

A 3-month-old infant is brought for evaluation of muscle stiffness with hyperextension of his neck and back. He has had failure to thrive with swallowing difficulties. There is concern about possible seizures. He is extremely irritable. Examination is notable for an irritable, cachexic infant with strabismus, bulbar dysfunction and hypertonia. There is hepatosplenomegaly. A previous child in the family had a similar presentation and died at age 4 years. Activity of which of the following enzymes is most likely to be abnormal? arylsulfatase galactocerebrosidase glucocerebrosidase hexosaminidase sialidase

Correct Answer is: glucocerebrosidase Gaucher disease is the most common lysosomal storage disease. It is caused by an autosomal recessive inherited deficiency of lysosomal glucocerebrosidase (gene locus 1q21). Non-neuronopathic (visceral) Gaucher disease was assigned as type I and neuronopathic glucocerebrosidase deficiency was referred to as type II (acute neuronopathic) or type III (subacute neuronopathic). Meanwhile, it has been recognized that all forms represent a broad spectrum of manifestations rather than clearly distinguishable disorders. Neuronopathic type is found in less than 1:100,000 life births. So far, more than 200 mutations have been described, but lack a genotype-phenotype correlation. Patients with acute neuronopathic Gaucher disease exhibit severe neurological symptoms during the first year of life including stridor, strabismus due to palsy of the ocular muscles, and dysphagia. Some patients show affection of the pyramidal tract with spasm, retroflexion of the head or opisthotonus. Acute neuronopathic Gaucher disease is usually fatal within the first two years of life.

A 3-month-old baby is brought to the emergency department by EMS 20 minutes after she stopped breathing and had a seizure at home. Her stepfather says that she rolled off of the couch onto the ground, cried briefly, then stopped breathing, became cyanotic and stiff. This lasted 5 minutes and then she became limp and unarousable. In the emergency room, her vital signs are stable. There is no external evidence of trauma. She does not arouse with vigorous physical stimulation. Anterior fontanelle is bulging and tight. Pupils are 5 mm and sluggish, oculocephalic reflexes are intact. Funduscopic exam shows bilateral retinal hemorrhages. Muscle tone and tendon reflexes are decreased. A CT scan is obtained, showing extraaxial effusions and a subdural hemorrhage in the right frontal area. Nonaccidental trauma is suspected. Which of the following metabolic conditions must be excluded as an alternative explanation? glutaric acidemia Leigh disease maple syrup urine disease Menkes disease neonatal adrenoleukodystrophy

Correct Answer is: glutaric acidemia The patient described here has a history, exam, and CT findings suggestive of shaken baby syndrome (SBS), a cause of nonaccidental head injury (NAHI). It is estimated that one-fourth to one-half of all children under 24 months of age admitted to the hospital for head trauma are victims of abuse. Because of the difficulty in categorizing head trauma as non-accidental outside of a perpetrator's confession, NAHI is often a diagnosis of exclusion. Shaken baby syndrome is one of the most difficult causes of NAHI to diagnose. External bruising has been shown to be absent in a significant minority (21%) of fatal NAHI cases and a history of previous maltreatment can be absent in up to 40% of all cases. SBS refers to a constellation of inflicted injuries mostly seen in young children (mean age about 4 months), caused by violent shaking, and is typically characterized by subdural hematomas and retinal hemorrhages. Retinal hemorrhages, however, have also been found in other conditions, including osteogenesis imperfect, homozygous protein C deficiency, Terson syndrome (intraocular bleeding associated with acute intracranial hemorrhage), Hermansky-Pudlak syndrome (an autosomal recessive condition characterized by albinism and platelet abnormalities), hemorrhagic disease of the newborn, and glutaric acidemia type 1

A 3-year-old boy is brought to the emergency room by the paramedics after having generalized convulsive seizure lasting 4 minutes. He has no history of prior seizures. His mother states that he has had 4 days of diarrhea, initially watery, but today it was bloody, and she had made an appointment with his pediatrician for this afternoon. General examination shows a low grade fever (38 degrees Celsius) and a mildly full abdomen. He has no rash or neurocutaneous stigmata. He is minimally arousable, only moaning with examination. Laboratory studies show a low hematocrit of 32%, a white blood cell count of 13.5/mm3, and a platelet count of 52,000/uL. His basic metabolic panel shows a BUN of 110, a creatinine of 2.1 mg/dL and glucose of 136 mg/dL. His urine shows moderate blood and protein, and hyaline casts Which of the following is the most likely diagnosis? celiac disease febrile seizure hemolytic uremic syndrome idiopathic thrombycytopenic purpura shigellosis

Correct Answer is: hemolytic uremic syndrome Hemolytic uremia has a classic triad of microangiopathic hemolytic anemia, thrombocytopenia, and acute renal failure. Two thirds of all cases occur in children younger than 3 years, with few cases after 5 years of age. In older adults it can occur, usually with an identifiable provocation. Neurological manifestations arise in approximately one third of all patients with HUS. Most of these tend to be mild and transient, but convulsions and ischemic cerebral infarctions, especially in the basal ganglia, can be severe

A 9-year-old boy is admitted for sudden headache, leading rapidly to depressed level of consciousness and weakness on the left side of the body. He had been watching television with his family. There was no recent history for fever, trauma, or systemic complaints. He was previously well and took no medications. On examination he was arousable with deep pain, but with gaze preference to his left and lack of motor response to the left arm or leg. CT imaging of the brain in the Emergency Department revealed a large intraparenchymal hemorrhage within the right parieto-occipital region with evidence of herniation of brain tissue from right to left under the falx. An emergency craniotomy was performed to evacuate the hemorrhage, and subsequent angiography demonstrated an arteriovenous malformation (AVM) in the right parietal midline region fed predominately by the right posterior cerebral artery. Which of the following symptoms would be most likely have been present in this child prior to rupture of the AVM? episodes of left sided weakness learning disability migraine with aura no symptoms prior to the hemorrhage simple partial sensory seizures

Correct Answer is: no symptoms prior to the hemorrhage After the newborn period, the most common cause of non-traumatic intraparenchymal brain hemorrhage in children is an AVM. For 75-80% of children and adolescents, there are no symptoms prior to the hemorrhage itself. Relative to bleeding from AVMs in adults, there is a higher likelihood of hemorrhage into the posterior fossa or deep cerebral structures, and the bleeding itself is often more massive. The morbidity and mortality with hemorrhage from AVMs in children is high, and a proactive approach to treatment is usually recommended when they are discovered. Surgical treatment should be individualized to the circumstances of each case, but can include craniotomy with surgical excision, endovascular embolization, or stereotactic radiosurgery (or some combination of these).

A 16-month-old boy is evaluated for microcephaly and developmental delay. He and his family migrated to the United States recently from a village in Bosnia. His mother had 2 other children (with different fathers) who both have mild mental retardation, microcephaly, and delay in linguistic development, congenital heart defects and severe growth retardation. The mother reports eating a "vegetarian" diet as a child but currently does not practice any dietary restrictions. She barely graduated high school and has difficulty remembering appointments and paying bills. Which of the following is the most likely explanation for this child's condition? chromosome disorder infantile autism Leigh's disease maternal phenylketonuria urea cycle disorder

Correct Answer is: maternal phenylketonuria Phenylketonuria (PKU) is an autosomal recessive disorder of amino acid metabolism affecting approximately 1/5000-10,000 infants in North America. It is most often due to deficiency of the enzyme phenylalanine hydroxylase which causes the accumulation of harmful metabolites, including phenylketones. If untreated, PKU leads to mental retardation, seizures, psychoses, eczema and a distinctive "mousy" odor. Pregnant women who have PKU and are not maintaining dietary restrictions are at risk of delivering infants with the effects of untreated maternal PKU and may also be at risk of developing neurologic symptoms themselves if they don't maintain lifelong dietary restrictions. The characteristic features of maternal PKU syndrome include mental retardation, microcephaly, intrauterine growth retardation, and congenital heart defects. The risk for these defects is increased with maternal plasma phenylalanine levels >20 mg/dL).

A 1-hour-old baby is transferred to the neonatal ICU after being born by emergency C-section at full term to a 32year-old woman. Pregnancy was uncomplicated until the day of delivery when vaginal bleeding was noted and investigation demonstrated placental abruption. Following emergent delivery, Apgar scores were 2 at 1 minute and 3 at 5 minutes. The baby was resuscitated and is on mechanical ventilation. Examination demonstrates decreased tone, minimal response to stimulation, sluggish pupillary, oculocephalic, and corneal reflexes. Which of the following treatments is most likely to minimize the likelihood of the future development of cerebral palsy? dextromethorphan indomethacin magnesium sulfate moderate hypothermia phenytoin

Correct Answer is: moderate hypothermia Disorders that damage the developing brain are a substantial cause of death or permanent disability such as cerebral palsy. Hypoxic-ischemic encephalopathy (HIE) in term infants occurs at a rate of about 3 per 1000 liveborn infants in developed countries, higher in the developing world. As many as a million deaths worldwide might be caused by perinatal asphyxia. Physicians who care for these children in the period after injury have traditionally provided supportive care with little expectation that their interventions would salvage brain tissue or have an effect on the final outcome. However, this situation has changed over the past 5 years with the publication of positive results from several controlled trials of therapeutic moderate hypothermia for term infants with neonatal HIE. Publication of these results has led to wider clinical application of therapeutic hypothermia for neonatal encephalopathy. These advances in treatment are based on knowledge gained from clinical observation of babies with asphyxia and extensive laboratory research with experimental models. Data from these experiments led to a heuristic model in which hypoxia-ischemia triggers a delayed series of events that lead to cell death in the brain. This period of delay, or latent interval, suggested that post-insult interventions could be protective if started in time. Studies show that moderate hypothermia within 6 h of asphyxia improves survival without cerebral palsy or other disability by about 40% and reduces death or neurological disability by nearly 30%

A 2-year-old girl is seen in the Emergency Department for altered consciousness. Her family found her in the living room, pale, disoriented, and unable to walk. She has become progressively more lethargic, and is barely arousable at the time of her arrival at the hospital. She has had no recent illnesses or injuries and has had normal development. On examination she is normotensive and afebrile, but mildly tachycardic. She reacts to deep pain with grimace and withdrawal movements, but without eye opening. All four extremities appear to move equally. Her examination is most notable for pinpoint pupils. Further history with the family indicates that the child's grandfather (also living in the same home) is on oxycodone hydrochloride for chronic pain. According to the child's mother, her father-in-law has a bad habit of leaving his medication around the house. Which of the following medications should be administered? apomorphine clonidine flumazenil naloxone propranolol

Correct Answer is: naloxone Opiate overdoses are characterized by depressed consciousness, decreased respiratory drive, and miosis, and can lead to hypothermia, bradycardia, and hypotension if sufficient quantities are ingested. Respiratory support may be required. Naloxone reverses the pharmacological effects of opiates by binding to opiod receptors (specifically with preferential binding to the μ-opiod receptor) as a competitive antagonist. When injected intravenously, it acts quickly (one minute), but has a relatively short duration of action (less than one hour), and so additional doses may be required to counter-act the opiate effect

A 14-year-old boy was admitted with severe headache leading to unconsciousness. Head CT in the Emergency Department revealed subarachnoid hemorrhage, with a large amount of blood in the cisterns at the base of the brain. There did not appear to be any evidence for intraparenchymal hemorrhage or ischemic stroke. The patient had a history of participating in skateboarding competitions. He is well-known locally for attempting risky jumps that are avoided by other skateboarders. He has had multiple falls in the past, but has never been knocked unconscious. He is otherwise healthy. There is no history for substance abuse, and toxicology results at the time of hospital admission are negative. An angiogram reveals an aneurysm of the supraclinoid portion of the right internal carotid artery, perhaps secondary to earlier traumatic injury. In order to prevent secondary complications from vasospasm in this patient, which of the following medications should be administered? clonidine heparin nimodipine phenytoin propranolol

Correct Answer is: nimodipine Ruptured intracranial aneurysms are relatively rare in the patients less than 18 years of age. In contrast to adult patients, in whom sporadic saccular aneurysms predominate, aneurysms in children are more diverse, including giant, fusiform, dissecting or infectious aneurysms. There is a greater likelihood of involvement of the posterior cerebral circulation. Approximately 10% are related to a familial predisposition. Even with severe subarachnoid hemorrhage (SAH), the prognosis in children may be more favorable relative to that seen in adults. Multiple complications following SAH are possible, including hydrocephalus, seizures, syndrome of inappropriate antidiuretic hormone (SIADH) or vasospasm leading to ischemic infarction. Nimodipine is approved for prevention of vasospasm in this circumstance

A 2-month-old is being evaluated for severe hypotonia. Pregnancy and delivery were unremarkable, but he was noted to be hypotonic at birth. On examination, he has a flat occiput, frontal bossing, large fontanelles, epicanthal folds, shallow supraorbital ridges and hepatomegaly. Which of the following tests is most likely to be diagnostic? chromosome analysis CT scan of the brain plasma carnitine levels plasma very long chain fatty acid levels skeletal X-rays

Correct Answer is: plasma very long chain fatty acid levels Peroxisomes are cytoplasmic organelles in all mammalian cells except RBC; single membrane. They contain no nucleic acid; the proteins are encoded by nuclear genes and translated on free cytoplasmic ribosomes. Peroxisomes perform a variety of cellular anabolic and catabolic functions including degradation of very long chain fatty acids (C22, C24, C26), early steps of plasmalogen biosynthesis, degradation of phytanic acid, selected steps in cholesterol biosynthesis, degradation of pipecolic acid, synthesis of bile acid intermediates, and glyoxylate metabolism. Peroxisomal disorders result from failure to form or maintain the peroxisomes or a defect in one of the enzymes normally found in the peroxisome. Zellweger syndrome is due to total absence of liver peroxisomes. Patients present with dysmorphic facies, hypotonia, seizures, mental retardation, neuronal heterotopias, cataracts and/or glaucoma, and renal cysts; about 50% have epiphyseal calcifications. Death usually occurs by 6-12 months. The diagnosis depends on showing lack of all peroxisomal function. No peroxisomes would be seen on EM. Elevated VLCFA or low plasmalogens are good screening tests.

A 2-month-old is being evaluated for severe hypotonia. Pregnancy and delivery were unremarkable, but he was noted to be hypotonic at birth. On examination, he has a flat occiput, frontal bossing, large fontanelles, epicanthal folds, shallow supraorbital ridges and hepatomegaly. Which of the following tests is most likely to be diagnostic? chromosome analysis CT scan of the brain plasma carnitine levels plasma very long chain fatty acid levels skeletal X-rays

Correct Answer is: plasma very long chain fatty acid levels Peroxisomes are cytoplasmic organelles in all mammalian cells except RBC; single membrane. They contain no nucleic acid; the proteins are encoded by nuclear genes and translated on free cytoplasmic ribosomes. Peroxisomes perform a variety of cellular anabolic and catabolic functions including degradation of very long chain fatty acids (C22, C24, C26), early steps of plasmalogen biosynthesis, degradation of phytanic acid, selected steps in cholesterol biosynthesis, degradation of pipecolic acid, synthesis of bile acid intermediates, and glyoxylate metabolism. Peroxisomal disorders result from failure to form or maintain the peroxisomes or a defect in one of the enzymes normally found in the peroxisome. Zellweger syndrome is due to total absence of liver peroxisomes. Patients present with dysmorphic facies, hypotonia, seizures, mental retardation, neuronal heterotopias, cataracts and/or glaucoma, and renal cysts; about 50% have epiphyseal calcifications. Death usually occurs by 6-12 months. The diagnosis depends on showing lack of all peroxisomal function. No peroxisomes would be seen on EM. Elevated VLCFA or low plasmalogens are good screening tests.

A 4-year-old boy is being seen in follow-up for cerebral palsy. He was born prematurely at 32 weeks gestation and had both hypoxic ischemic injury and grade 3 intraventricular hemorrhage requiring ventriculoperitoneal shunting. He has been getting physical therapy 3 times weekly and is on baclofen 10 mg four times daily. Despite these measures, he continues to have increased tone and often appears uncomfortable because of dystonic muscle contractions which cause twisting of his limbs and lateral torsion of his trunk. On examination, he has brisk deep tendon reflexes and increased tone in all limbs, but spasticity is minimal. Which of the following is the most appropriate next step in his management? assess for shunt malfunction obtain an EEG prescribe dantrolene prescribe diazepam prescribe trihexyphenidyl

Correct Answer is: prescribe trihexyphenidyl Cerebral palsy is the most common motor disability in childhood. Abnormalities in muscle tone are typically present, with low tone in some muscle groups and increased tone in others. Hypertonia is defined as abnormally increased resistance to externally imposed movement about a joint. Hypertonia may be caused by spasticity, dystonia, and rigidity, alone or in combination. It is important to distinguish between these syndromes because each may require different treatments. Spasticity is the most common tone abnormality in children with cerebral palsy. It is a velocity-dependent resistance of a muscle to stretch and is defined as having one or both of the following: (1) resistance to externally imposed movement increases with increasing speed of stretch and varies with the direction of joint movement, and/or (2) resistance to externally imposed movement rises rapidly above a threshold speed or joint angle. Upper motor signs (hyperreflexia, clonus, Babinski response) are commonly present on exam. Children who continue to have problems with hypertonia despite adequate spasticity treatment may have dystonia or rigidity. The patient described here has the typical features of dystonia, in which cocontraction of agonist and antagonist muscle groups result in twisting and abnormal postures. In the child whose etiology is undetermined, a trial of carbidopa-levodopa is given, as some have "dopa-responsive dystonia". Otherwise, trihexyphenidyl is used, starting at 2.5 mg/day and titrating up slowly to 60-80 mg/day.

A 10-year-old boy is admitted with new onset status epilepticus. He was seen by his pediatrician 3 days prior to admission for a large, tender lymph node in the right neck. A 10 day course of Augmentin was prescribed, and he was told to follow up in one week. On the morning of admission his mother had difficulty getting him to arouse and brought him to the ER. In the ER he developed right face and arm clonic activity, which stopped after he was given 3 doses of lorazepam and a loading dose of fosphenytoin. Thereafter, an EEG showed intermittent non-convulsive electrographic seizures from the left hemisphere. CT scan of the brain was normal. A lumbar puncture revealed a white blood count of 18 cells/mm3, with normal glucose and protein levels. The patient was started on vancomycin, ceftriaxone and acyclovir. Herpes simplex PCR testing was negative. Which of the following laboratory tests is most likely to be abnormal? acid fast bacilli culture of the cerebrospinal fluid cerebrospinal fluid for cryptococcal antigen repeat cerebrospinal fluid PCR for HSV I/II serum Bartonella henselae titers serum ELISA for Borrelia burgdorferi

Correct Answer is: serum Bartonella henselae titers Bartonella infections are most often acquired by a recent inoculation through a skin abrasion or scratch from a young kitten. Classically, affected patients demonstrate prominent lymphadenopathy in the cervical, axillary or inguinal region. Other systemic symptoms such as fever, anorexia, fatigue and headache may also be present. Atypical cases of cat scratch disease do occur, often with neurologic features. Encephalopathy (presenting with mental status changes or new-onset seizures, often status epilepticus) is the most common neurologic presentation, representing about 2% of cases of cat scratch disease in the largest case series reviewed. Less likely neurologic presentations include acute or subacute monocular blindness due to retinitis, a peripheral neuritis resembling Guillain-Barre syndrome, meningitis and a transverse myelitis. Fever is not necessarily present during the encephalopathy and cerebrospinal fluid exam may not show pleocytosis. Hyperintense lesions can be seen on T2-weighted and FLAIR MRI in the deep grey structures and rarely the cortex, though imaging studies are typically normal. A high degree of suspicion for the disease should be maintained in anyone with acute or subacute encephalopathy and either lymphadenopathy and/or recent kitten exposure. Although multiple antibiotic regimens have been utilized in patients with focal brain lesions or encephalitis from bartonella infections, their benefit is uncertain. Spontaneous resolution is the rule for this disease, though rarely chronic epilepsy may develop. Lyme disease can present with a number of neurologic features, though seizures and status epilepticus are not common. Subacute or chronic meningitis and unilateral or bilateral facial nerve palsies are the most common neurologic presentations in children. Rarely, initial PCR testing for herpes simplex encephalitis can have a false negative result. In cases with a high index of suspicion, especially with focal seizures and hemorrhagic infarctions in the temporal or orbital frontal cortices, repeat PCR testing should be considered. Both cryptococcal and M. tuberculosis infections of the central nervous system would typically present in an immune compromised host. Although atypical mycobacterial infections can lead to prominent cervical lymphadenopathy, this feature is not present in M. tuberculosis infection

A 15-year-old girl is being evaluated for headaches which began about a year ago. They occur 3-4 days per week, are located in the occipital area, and last for several hours unless she takes ibuprofen. Sometimes they are triggered by coughing or sneezing. She denies associated nausea, vomiting, sensitivity to light or sound, and dizziness but says sometimes objects look like they are jiggling when she has the pain. Her past medical history is otherwise unremarkable and family history is noncontributory. Examination is normal except for hyperreflexia in the lower extremities Which of the following secondary complications is most likely to develop if her condition progresses? hemicord syndrome spinal cord infarction spinal stenosis syringomyelia transverse myelitis

Correct Answer is: syringomyelia The patient presented here demonstrates a Chiari I malformation. This condition was first described by Hans Chiari in 1891. It is defined as cerebellar tonsil extension below the foramen magnum. Severity ranges from a morphological asymptomatic finding to a clinical Chiari. There appears to be evolution of the condition over time, the number of symptomatic patients increasing with age. Patients with this malformation are at risk of developing a dilation of the central canal (syrinx or hydromyelia). Abnormal CSF flow at the level of the foramen magnum is due to underdevelopment of the mesodermal occipital somite, resulting in a small volume of the posterior fossa. How this results in dilation of the central canal is not well understood. Nonetheless, treatment of the condition is typically neurosurgical decompression of the posterior fossa via a suboccipital craniectomy, often combined with laminectomy to the level of the tonsillar herniation, with duraplasty or dural scoring. Surgical complication rate is much higher if the subarachnoid membrane is invaded.

A seven-year-old boy with a diagnosis of intractable epilepsy is prescribed an anticonvulsant. Six months later, he presents with a four hour history of colicky abdominal pain and vomiting. Examination reveals him to be distressed and pale. He is not jaundiced. During the examination he suffers a bout of pain. A diagnosis of renal colic is made. Which of the following anticonvulsants is most likely to be associated with this clinical scenario? carbamazepine lamotrigine phenytoin sodium valproate topiramate

Correct Answer is: topiramate Long-term use of topiramate (TPM) may result in increased incidence of asymptomatic kidney stones in the pediatric population. The result of a retrospective observational study showed that 5.2% of the total sample had developed asymptomatic kidney stones. Moreover, the prevalence (5.2%) of the development of kidney stones in children during use of TPM was significantly higher when compared to the prevalence of 0.367%. Hence, routine baseline and follow-up ultrasound of the urinary system should be recommended during the use of TPM in children.


Related study sets

Florida statutes, rules, and regulations (all lines)

View Set

MICROECON: CHAPTER 12 "LABOR MARKET"

View Set

Organizational Motivation and Leadership Final Exam

View Set